BarBri Real Property MBE

अब Quizwiz के साथ अपने होमवर्क और परीक्षाओं को एस करें!

A father drew up a deed conveying his land to his son. The father never recorded the deed and left it in the top drawer of his desk in his study. Two years later, the father died. He left a will, which declared that all of his property be divided equally between the son and the father's daughter. While going through his father's personal effects, the son discovered the deed to the land. He showed the deed to his sister and the two of them agreed not to record the deed. The son put the deed in a desk drawer in his home. A year later, the son died. As the executor perused the son's personal papers, he came across the deed and promptly recorded it. He then entered into a contract to sell the land to a buyer. The daughter discovered this and promptly filed suit, claiming an interest in the land. A statute of the jurisdiction provides: "No conveyance is good against a subsequent purchaser for value, without notice, who first records." How will the court rule? (A) In favor of the daughter, because there was no proper delivery of the deed to the land. (B) In favor of the daughter, because the executor violated his fiduciary duty when he recorded. (C) Against the daughter, because the executor and the buyer are protected by the recording act. (D) Against the daughter, because she is not a bona fide purchaser.

(A) The daughter will prevail because the deed executed by the father to the son was never properly delivered. A deed is not effective unless it is delivered. Unless there is some clear expression of intent that the grantor envisioned the passage of title to the grantee (i.e., that the grantor intended to relinquish control over the property), the continued possession of the deed raises a presumption of nondelivery. The father did not do anything to indicate an intent to pass immediate title to the son. The presumption of nondelivery is not rebutted and the father retained title. Therefore, the land was part of the father's estate and the daughter and the son inherited it as tenants in common. (Multiple grantees are presumed to take as tenants in common.) The daughter has an undivided one-half interest in the land. (B) is wrong because whether the executor violated his fiduciary duty has no bearing on the ownership of the land. That fact would be relevant only if the daughter were seeking damages from the executor. The executor's recording of the void deed did not damage the daughter because it did not affect her rights. (C) is wrong for two reasons: (i) The buyer cannot be protected by the recording act because he has not yet recorded and this is a race-notice jurisdiction. To prevail in a race-notice jurisdiction, a party must be a subsequent purchaser for value, without notice of an adverse claim, and must record first. The buyer has not yet recorded. It is not clear from the facts whether he has even received the deed. If he has not, he cannot qualify as a bona fide purchaser because he now has notice of the daughter's claim. (ii) The executor cannot claim the protection of the recording act because he is acting as the son's agent. The son was a donee with notice and as such was outside the protective provisions of the recording act. (D) is wrong because, as explained above, the daughter need not be a bona fide purchaser (i.e., need not turn to the recording act) to prevail. She inherited an undivided one-half interest in the land, which has not been cut off by the subsequent acquisition and recording by a bona fide purchaser.

A developer and an investor had been in the real estate business for many years. Because of their long-standing relationship, the developer and the investor, neither of whom was an attorney, often dispensed with certain legal formalities when dealing with each other, thus saving the costs of lawyers' fees and other attendant expenses. The investor owned a parcel of land that the developer was interested in. At lunch one day, the developer offered to buy the parcel from the investor for $50,000. The investor accepted the developer's offer, and the parties agreed on June 15 as the closing date. The developer wrote out and handed the investor a check for $2,500 with "earnest money" written in the memo, and they shook hands on their deal. A few weeks before closing, the developer called the investor and told him she had changed her mind about purchasing the land because of a sudden economic downturn in the area. The investor appeared at the developer's office on June 15 with the deed to the land in his hand. The developer refused to tender the balance due, and the investor sued the developer for specific performance. Will the investor prevail? (A) No, because the agreement does not comply with the Statute of Frauds and is, therefore, unenforceable. (B) No, but the court will allow the investor to keep the $2,500 earnest money as damages. (C) Yes, because the $2,500 payment constituted part performance of the contract. (D) Yes, because the developer and the investor had established a course of dealing.

(A) The investor will not succeed in a suit for specific performance because the agreement is unenforceable under the Statute of Frauds. Under the Statute of Frauds, a land sale contract is unenforceable unless it is in writing and signed by the party to be charged. The Statute of Frauds requires the writing to contain all essential terms of the contract, which are: (i) a description of the property, (ii) identification of the parties to the contract, (iii) the price, and (iv) the signature of the party to be charged. Here, the agreement between the investor and the developer concerns the sale of land; thus, the agreement must be evidenced by a writing to comply with the Statute of Frauds. The only writing mentioned in the facts is the check given to the investor by the developer. This check contains neither a description of the property that is the subject of the agreement nor the price. Thus, the check is not a writing sufficient to satisfy the Statute of Frauds. Consequently, the agreement is unenforceable, and the investor will not prevail. (B) is incorrect because, if there is no enforceable agreement, there can be no "breach" of the agreement, for which breach the investor would be entitled to damages. Therefore, the investor may not keep the earnest money as damages. (C) is incorrect. The doctrine of part performance generally requires two of following: possession, improvements, and full or partial payment of the purchase price. A few states will grant specific performance of a contract despite the absence of a writing if there has been payment of the purchase price. Even under this view, however, the developer's payment of $2,500 out of a total price of $50,000 will not constitute sufficient performance to remove this agreement from the purview of the Statute of Frauds. (D) is incorrect because "course of dealing" (i.e., a sequence of previous conduct between the parties that may be regarded as establishing a common basis of their understanding) may be used to explain or supplement the terms of a written contract under the Uniform Commercial Code ("UCC"). This question does not involve the sale of goods, so the UCC is inapplicable. Furthermore, here there is no written agreement, the terms of which can be explained or supplemented by showing a course of dealing between the developer and the investor. Although the developer and the investor often dispensed with legal formalities as a cost-saving measure, this "course of dealing" will not confer validity on their oral agreement for the sale of land.

A seller contracted to convey her property to a buyer for $75,000. A title search revealed the following: (i) There were 25 years left on a lease of the property, which was recorded. The buyer agreed to take title subject to the lease but was not aware that the lease gave the lessee, his heirs, and assigns an option to purchase the land at any time before the end of the lease period. (ii) The roof of the garage on the property extended approximately one-half inch across the property line into the airspace of an adjoining neighbor. The garage did not interfere with any current or future use of the adjoining lot. (iii) The home on the property was subject to a $5,000 lien arising from a dispute involving some remodeling work. The seller promised to pay off the lien at closing with the proceeds from the sale. (iv) The property was subject to an easement by necessity in favor of the adjoining neighbor. Last month, the city extended the main road to the neighbor's land, but the neighbor planned to continue to use the easement because it was more convenient. In a jurisdiction that has a standard racenotice recording statute and maintains the common law Rule Against Perpetuities without any modern statutory reformation, which encumbrance renders the seller's title unmarketable? (A) The lessee's option. (B) The encroachment of the garage's roof. (C) The $5,000 lien. (D) The easement.

(A) The lessee's option to purchase renders the seller's title unmarketable. Ordinarily, an option that is not personal to the holder (that is, an option that extends to the holder's heirs and assigns) would violate the Rule Against Perpetuities and be stricken. An interest violates the Rule Against Perpetuities if there is any possibility, however remote, that it will vest more than 21 years after some life in being at the creation of the interest. Here, the relevant measuring life would be the lessee. The lessee could die within the next three years, and the option could be exercised by his successors more than 21 years after his death. There is a special exception to the Rule, however, for options to purchase attached to leaseholds. Because the one who holds the option in this case is the current lessee, the Rule does not apply. Thus, the option is valid, and it renders the seller's title unmarketable. (B) is incorrect because only a significant encroachment will render title unmarketable. A one-half-inch encroachment on airspace would not be considered significant, particularly because it does not interfere with the use of the adjoining property. Thus, this encroachment will not affect marketability. (C) is incorrect because a lien on property will not render title unmarketable if the seller pays the lien at closing. Unless the contract provides otherwise, the seller need not provide marketable title until closing. A seller has the right to satisfy a lien at the closing with the proceeds of the sale. Therefore, as long as the purchase price is sufficient and the lien is satisfied simultaneously with the transfer of title (e.g., by using escrows), the buyer cannot claim that the title is unmarketable. The closing will result in marketable title. In this situation, the $75,000 purchase price is clearly sufficient to satisfy the $5,000 lien. Thus, the seller may satisfy the lien at the closing and convey marketable title to the buyer. (D) is incorrect because although an easement that reduces the value of the property renders title unmarketable, this easement has been terminated. An easement by necessity terminates as soon as the necessity ends. Because the main road now provides access to the neighbor's land, she no longer needs the easement over the seller's land and it is extinguished.

A businessman entered into a contract to sell his office complex to a purchaser for $1 million. The purchaser paid the businessman $100,000 in earnest money. The day before the date set for closing, the purchaser died intestate, leaving her niece as her only heir. The niece showed up at the closing with a certified check for $900,000. Which of the following is correct? (A) The niece may specifically enforce the agreement. (B) The businessman may return the $100,000 down payment and cancel the contract. (C) Death terminates the agreement. (D) Any title acquired would be unmarketable by reason of the purchaser's death.

(A) The niece may specifically enforce the agreement. Under the doctrine of equitable conversion, if the buyer dies, the takers of her real property can demand a conveyance of the land at the closing of the contract. (B) is wrong because the death of either the seller or the buyer does not render the agreement cancellable at the will of either party. (C) is wrong because a real estate contract survives the death of either party unless the agreement itself provides otherwise. (D) is wrong because marketable title is title reasonably free from doubt. Generally, this involves either defects in the chain of title or encumbrances that might present an unreasonable risk of litigation. Such problems are not present in these facts.

A buyer entered into a written contract to purchase a seller's house for $250,000. The contract called for the seller to deposit a deed in escrow forthwith, and for payment of the purchase price and delivery of the deed through escrow within 30 days thereafter. The seller immediately deposited the deed with the escrow holder. On the 29th day, the seller was injured in a snowmobile accident and rendered comatose; he remains in this state to date. On the 30th day, the sale closed pursuant to the contract; the seller's deed was delivered to the buyer, and the $250,000 was paid over to the seller's account. Which of the following is true of the seller's incapacity? (A) It has no effect on the buyer's title. (B) It constitutes a defect in the buyer's title. (C) It allows his court-appointed guardian to set aside the buyer's deed. (D) It prevented passage of title to the buyer.

(A) The seller's incapacity has no effect on the buyer's title. Generally, in an escrow transaction, title does not pass to the grantee until performance of the specified conditions (e.g., the payment of money). When a deed is placed in escrow, there is a valid conditional delivery, and title will transfer automatically on the occurrence of the condition. However, if justice requires, the title of the grantee is deemed to relate back to the time of the deposit of the deed in escrow. One situation in which the relation-back doctrine applies is when the grantor becomes incompetent (to avoid the rule that an incompetent cannot convey title). Here, title would ordinarily not have passed to the buyer until the purchase price was paid, at which time the deed would be delivered through escrow. However, at the time the money was paid and the deed was delivered to the buyer, the seller was comatose and incompetent to actually convey title. This situation would lead to the unjust result that the buyer would be prevented from taking title simply due to the unfortunate circumstances of the seller's sudden incapacitation. Such a result triggers application of the relation-back doctrine, so that title is deemed to have passed to the buyer at the time the seller deposited the deed in escrow. Therefore, the seller's incapacity has no effect on the buyer's title to the house. (D) is incorrect because the prevention of passage of title is precisely the result designed to be avoided by application of the relation-back doctrine. (B) and (C) both incorrectly assume that the seller's incapacity somehow interferes with or prevents the passage of title to the buyer. As explained above, pursuant to "relation back," the buyer has clear title going back to the time at which the deed was deposited with the escrow holder, prior to the seller's incapacity. Thus, Real Property Answers 7. there is no basis for concluding (as does (B)) that the seller's incapacity constitutes a defect in the buyer's title, or that (as (C) states) the seller's guardian is entitled to set aside the deed.

A vendor sold his house and lot to a vendee for $60,000 by a written agreement that called for a $10,000 down payment and $10,000 a month on the first of each month thereafter until the balance was paid. The vendee made the down payment and first month's payment on time. He made the second and third months' payments on the 15th of each month, skipped the fourth month entirely, and resumed payments on the 5th of the fifth month. The following week, the vendor filed an unlawful detainer action to have the vendee ousted and the contract forfeited. Who is likely to prevail? (A) The vendee, because the vendor waived the strict performance of the contract. (B) The vendee, if the contract did not provide that time was of the essence. (C) The vendor, but he will have to refund the vendee the amount of payments made. (D) The vendor, and he may retain the amount of payments made.

(A) The vendee is likely to prevail because the vendor waived the strict performance of the contract. In an installment land contract, the vendee signs a contract with the vendor, agreeing to make regular installment payments until the full contract price has been paid. Only at that time will the vendor give a deed transferring legal title to the purchaser. In case of default, the contract usually contains a forfeiture clause providing that the vendor may cancel the contract, retain all money paid to date, and retake possession of the land. However, where a vendor has established a pattern of accepting late payments from the vendee, he cannot suddenly insist on strict on-time payment and declare a forfeiture if such payment is not forthcoming. Such a pattern is said to constitute a waiver of strict performance. Here, the vendor accepted the vendee's late payments for at least three months. Thus, he is deemed to have waived the strict performance of the contract. (B) is incorrect because even if the contract provided that time was of the essence (i.e., that payments had to be made on the first of the month), the vendor will be deemed to have waived this provision by accepting the vendee's late payments. (C) is incorrect because the vendor will not prevail; if he did, however, he might have to refund the vendee a portion of the payments made based on an equitable claim for unjust enrichment by the vendee (e.g., at the very least, the fifth payment). (D) is incorrect because the vendor waived the strict performance of the contract, and it is also unlikely that the vendor would be able to retain all of the payments made, even if he were able to oust the vendee.

A buyer purchased a house from a seller. It turned out that the concrete used to pour the foundation had been improperly mixed and the foundation was crumbling. The buyer discovered that the cost of repairing the defective foundation would be over $10,000. She filed suit against the seller for the cost of repairs. If the court rules in the buyer's favor, what is the likely reason? (A) The crumbling foundation makes the house unsafe or uninhabitable. (B) The seller was the builder of the house. (C) The buyer took title to the house by warranty deed. (D) The buyer had no knowledge of the defect when she purchased the house, and the defect was not reasonably apparent.

(B) If the court rules in the buyer's favor it will be because the seller was the builder of the house. Generally, a conveyance of real property contains no warranties of quality or fitness for the purpose intended, but there is a recognized exception for the sale of a new house by the builder. There is an implied warranty that the new house is designed and constructed in a reasonably "workmanlike" manner and suitable for human habitation. Thus, in this case, the buyer would appear to have no claim against the seller unless he was the builder, in which case she could claim that the house was not constructed in a reasonably "workmanlike" manner. (A) is wrong because the fact that the house was unsafe and uninhabitable would not by itself result in the seller's 4. Real Property Answers liability unless he was the builder. (C) is wrong because the covenants contained in a warranty deed are covenants for title (i.e., they protect the purchaser against competing claims for the title to the property); they offer no protection against defects on the property. (D) is wrong because the buyer's knowledge of the defect is not relevant to the seller's liability under these facts. While a seller may be liable if he purposely conceals defects on the property or, in many states, if he does not disclose serious defects that he is aware of, he is not generally liable for defects in the absence of these circumstances. Because the facts do not indicate that the seller acted to conceal the crumbling foundation, or was even aware of the problem, he will not be liable for the conditions unless he was the builder.

A chef purchased a restaurant for $100,000. As part of his financing, he obtained a purchase money mortgage from a bank for $60,000. Due to a clerical error by the bank, the mortgage was not recorded in the county recorder's office. A statute in the jurisdiction provides: "No conveyance of an interest in land, other than a lease for less than one year, shall be valid against any subsequent purchaser for value, without notice thereof, whose conveyance is first recorded." After the chef's restaurant had been in operation for five years, business dropped dramatically. To stay in business, the chef obtained a mortgage from a financing company for $30,000. The financing company was not informed by the chef of the mortgage held by the bank. The next day, the chef contacted the bank about renegotiating its mortgage. Checking its records, the bank discovered that the original mortgage was not recorded and immediately recorded it. Later that day, the financing company recorded its mortgage. A few days later, the chef and the bank agreed to a modification of their mortgage agreement to allow the chef to make lower monthly payments in exchange for a higher interest rate and a longer period of repayment. Despite this agreement, the chef was unable to make payments on the financing company mortgage. The financing company instituted a foreclosure action six months later, but failed to include the bank as a party to the foreclosure action. If the financing company takes title to the restaurant at the foreclosure sale, which of the following statements most correctly describes the bank's interest? (A) The bank's mortgage on the restaurant survives under its original terms. (B) The bank's mortgage on the restaurant survives under its modified terms. (C) The bank's mortgage is extinguished because when it was modified it became junior to the financing company's mortgage. (D) The bank's mortgage is extinguished regardless of the modification because it had not recorded before the financing company obtained its mortgage interest.

(B) The bank's mortgage survives under its modified terms because the financing company did not include the bank in the foreclosure action. The general rule is that when a mortgage is foreclosed, the buyer at the sale will take title as it existed when the mortgage was placed on the property. Thus, foreclosure generally will destroy all interests junior to the mortgage being foreclosed, but will not discharge senior interests. However, those with interests subordinate to those of the foreclosing party are necessary parties to the foreclosure action. Failure to include a necessary party results in the preservation of that party's interest despite foreclosure and sale. Here, the bank's original mortgage was senior to the financing company's mortgage. However, where the landowner enters into a modification agreement with the senior mortgagee, raising its interest rate or otherwise making it more burdensome, the junior mortgagee will be given priority over the modification. Thus, the bank's modification would not have priority over the financing company's mortgage. Nevertheless, because the financing company failed to include the bank in its foreclosure action, the bank's mortgage interest survives under its modified terms, even though the modification did not have priority. (B) is therefore correct and (A) is incorrect. (C) is incorrect because the modification of a senior mortgage does not nullify its original senior status; it only means that the junior mortgage will be given priority over the modification. Because the buyer at the foreclosure sale ordinarily will take title as it existed when the mortgage was placed on the property, the senior mortgage ordinarily survives in its original form. (As noted above, here the mortgage survives in its modified form despite its junior status because of the failure to include the bank in the foreclosure action.) (D) is incorrect because the recording statute applicable here is a race-notice statute rather than a notice statute. Under a race-notice statute, a subsequent bona fide purchaser (including a mortgagee) is protected only if it records before the prior grantee or mortgagee. Here, the bank recorded its mortgage before the financing company recorded its mortgage. The fact that the financing company had no notice of the bank's interest at the time it granted the mortgage on the restaurant does not help the financing company because it did not record first.

A woman and her friend lived together in the woman's home for 20 years. Subsequently, the woman became disabled because of a heart ailment and the friend had to take care of her. The woman told the friend that she wanted to be sure that the friend got her house after she died, so she gave the friend a quitclaim deed. The friend did not record the deed, but put it in his safe deposit box. Four months later, the woman's son found out about this and told his mother that if she would sell the house to him, she could live there for the rest of her life. The woman, who wanted the money, agreed and carried out the transaction. She told the friend that she had changed her mind and decided to leave the home to her children. The friend promised to destroy the deed, and the next day, he did. Several days later, however, as the friend and the woman were driving to the store, their car was hit by a train and they both died. Both died intestate, and the son was the woman's sole heir. The son and the friend's heirs claim title to the house. The friend's heirs bring an appropriate action to resolve the dispute. Which party will the court likely find owned the house? (A) The friend's heirs, because the woman did not tell the friend the truth about why she was revoking her agreement. (B) The friend's heirs, because the friend did not retransfer title to the woman. (C) The son, because the friend agreed to return the title, and did in fact destroy the deed. (D) The son, because the friend, as a donee, would not be able to prevail against the son, who was a bona fide purchaser.

(B) The court will most likely find that the friend was still the owner of the house because he did not retransfer title to the woman. The deed, once delivered, merely evidences title to the property, and its destruction has no effect on the title. Thus, the friend was still the owner, and (C) is incorrect. (A) is incorrect because the woman cannot affect the ownership of the property after she delivered the deed to the friend, regardless of the truth or falsity of her subsequent reasons for the attempted revocation. (D) is incorrect because the son was not a bona fide purchaser: He was aware of the friend's deed. Also, it is only the subsequent bona fide purchaser, a purchaser for value and without notice, who gets the protection of the recording act. The friend was the first grantee and, thus, his status as a donee would be irrelevant to the applicability of the recording act.

To satisfy a debt owed to a creditor, a son executed and delivered to the creditor a warranty deed to a large tract of undeveloped land. The creditor promptly recorded the deed. Shortly thereafter, she built a house on the property and has lived there ever since. The son never actually owned the land. It belonged to his father, but the father had promised to leave the property to the son. Later, the father died and his will devised the property to the son. Pressed for money, the son then sold the land to an investor by warranty deed, which the investor promptly recorded. Although the investor paid full value for the property, he purchased it strictly for investment and never visited the site. He therefore did not realize that the creditor was living there, and knew nothing of the son's earlier deed to the creditor. The jurisdiction in which the land is located has the following statute: "A conveyance of an estate in land (other than a lease for less than one year) shall not be valid against any subsequent purchaser for value without notice thereof unless the conveyance is recorded." Which of the following is the most likely outcome of a quiet title action brought by the creditor against the investor? (A) The creditor prevails, because the son had no title to convey to the investor. (B) The creditor prevails, because the investor was not a purchaser for value without notice of the creditor's interest. (C) The investor prevails, because under the doctrine of estoppel by deed, title inures to the benefit of the original grantee only as against the grantor. (D) The investor prevails, because under the recording acts, the deed from the son to the creditor was not in the chain of title and hence did not constitute notice to the investor.

(B) The creditor will prevail in a suit to quiet title because the investor had notice of the creditor's interest in the property and, thus, is not a bona fide purchaser for value. When a grantor purports to convey property that he does not own, his subsequent acquisition of title to that property vests in the grantee under the doctrine of estoppel by deed. Most courts, however, hold that this is personal estoppel, which means that title inures to the grantee's benefit only as against the grantor, not a subsequent bona fide purchaser. If the grantor transfers his after-acquired title to an innocent purchaser for value, the bona fide purchaser gets good title. There is a split of authority as to whether the original grantee's recordation of the deed imparts sufficient notice to prevent a subsequent purchaser from being a bona fide purchaser, but the majority view is that it does not because it is not in his chain of title. Thus, it is not the fact that the creditor recorded that prevents the investor from being a bona fide purchaser. The fact that the creditor built a home and was living on the property gave the investor constructive notice of her interest. A title search is not complete without an examination of possession. If the possession is unexplained by the record, the subsequent purchaser is charged with knowledge of whatever an inspection of the property would have disclosed and anything that would have been disclosed by inquiring of the possessor. Therefore, the investor is charged with knowledge of the creditor's possession and with what the creditor would have told him about her possession; i.e., that the property was conveyed to her by the son prior to his conveyance to the investor. Consequently, the investor does not qualify as a bona fide purchaser, and (C) is an incorrect choice. (A) is incorrect because, although the son is estopped to deny that he acquired title for the benefit of the creditor, he could have conveyed valid title to a subsequent purchaser for value who had no notice of the creditor's interest. Therefore, it is not exactly correct to say that the son had no title to convey. (D) is incorrect because the investor will not prevail. It is true that under the recording acts the creditor's deed was not in the chain of title, but the investor still does not qualify as a bona fide purchaser. The investor is on inquiry notice arising from the creditor's possession of the property.

A mother purchased 80 acres of desert land over 30 years ago. The deed was properly recorded. Although her family had never even visited the land, the mother had described to them the little two-room cabin that sat in the middle of the parcel near the dry streambed. Ten years ago, the mother's son found what he was certain was the little cabin, and over the next few years he built a barn, a greenhouse, and some corrals, all enclosed by a sturdy wire mesh fence. The area bounded by the fence, containing all the structures, occupied about two acres of the 80 owned by the mother. Three years ago, the mother died, validly devising the 80 acres to the son. The son entered into a contract for sale of the two acres, describing it in detail with reference to the structures and nearby landmarks. The purchaser's surveyor discovered that the son had settled onto a completely different parcel from the one owned by the mother. The purchaser immediately announced that he would not proceed with the sale contract. The state's statutory period for establishing adverse possession is five years. If the son brings an action for specific performance of the sale contract, for whom should the court rule? (A) The purchaser, because the son does not own the land he is purporting to sell. (B) The purchaser, because the son does not have marketable title to the land he is purporting to sell. (C) The son, if he conveys by quitclaim deed. (D) The son, because the description in the contract of sale is sufficient to identify the property and need not be as accurate as one contained in a deed conveying land.

(B) The purchaser prevails because the son's acquisition of the land by adverse possession does not satisfy his implied covenant to deliver a marketable title. Unless expressly provided otherwise, all contracts for the sale of land contain an implied covenant by the seller that he will deliver to the buyer at closing a marketable title. Title is marketable if a reasonably prudent buyer, ready and able to purchase, would accept it in the exercise of ordinary prudence. Generally, inability to establish a record chain of title will render title unmarketable. If a seller attempts to rely on adverse possession to establish marketable title, many courts will hold that such title is not marketable until the adverse possessor has perfected it by a judgment quieting title. In other words, the buyer is not required to "buy a lawsuit." Other states require only that the seller provide written evidence or some other proof that the buyer can use in court to defend any lawsuit challenging title. Even under the latter approach, it does not appear that the son can provide enough evidence to make his title marketable. Thus, although the son has acquired title to the land by occupying it in an open, notorious, hostile, and continuous manner, for a period exceeding that prescribed by statute, the fact that he has acquired title only by adverse possession renders his title unmarketable. Because the son has thus breached his covenant of marketable title, implied in the sale contract, his action to specifically enforce that contract will fail. (A) is incorrect because the son does own the two acres he is trying to sell. The son's fencing of the land and building structures on it qualifies as open and notorious possession because it is such as the usual owner would make of the land and puts the true owner or the community on notice of the fact of possession. His possession was continuous for more than the five-year statutory period, and it was hostile because it was without the permission of the true owner. Under the majority view, the son's good faith belief that he was possessing the land Real Property Answers 3. described by the mother's deed is irrelevant. Thus, the son acquired title by adverse possession. However, as explained above, such title, as far as the purchaser is concerned, is not marketable. (C) is incorrect because the type of deed by which title is transferred does not affect the seller's covenant of marketable title, which is implied in the contract. It is true that the implied covenant of marketability is no longer assertable once a deed has been delivered (absent fraud or mistake), so that if the son delivered a quitclaim deed (i.e., without making any assertions relative to the title being transferred), the purchaser could no longer assert the implied contractual covenant of marketable title. However, no deed has yet been delivered; the covenants under the contract are still in effect. (D) is incorrect because the fact that a contract describes land with sufficient specificity does not establish the marketability of title to that land.

To secure a loan of $100,000 from a bank, the owner in fee simple of a parcel of land conveyed a deed of trust for the land to the bank. The deed of trust contained a "power of sale" clause, permitted by the jurisdiction, which allowed the bank to sell the property in the event of default without the necessity of a judicial foreclosure action. After several years, the owner defaulted on his loan payments to the bank. The bank informed the owner that it was exercising its power of sale. After appropriate notices, the bank conducted a public sale of the land. The bank was the sole bidder and obtained the property for $80,000, which was $10,000 less than the outstanding balance on the loan plus the expenses of the sale. One month later, the owner notified the bank that he wanted to pay off the loan and extinguish the deed of trust, and was prepared to tender $80,000 to do so. The bank insisted that the owner must tender $90,000 to pay off the loan. If a court in the jurisdiction will require the bank to accept only $80,000 under the circumstances above, what is the likely reason? (A) The owner had the power to revoke the trust as long as he was alive. (B) The bank did not have the authority to bid on the property at other than a judicial foreclosure sale. (C) The owner was exercising a statutory power rather than an equitable power. (D) The bank does not have the power to clog the equity of redemption.

(C) If the owner can compel the bank to accept his offer, it will be because he has a statutory power to redeem the property after the foreclosure sale has occurred. In all states, the equity of redemption provides the borrower with an equitable right, at any time prior to the foreclosure sale, to redeem the land or free it of the mortgage or lien by paying off the amount due or, if an acceleration clause applies, the full balance due. Only about half the states, however, give the borrower a statutory right to redeem for some fixed period after the foreclosure sale has occurred; the amount to be paid is generally the foreclosure sale price, rather than the amount of the original debt. Thus, if the owner can redeem the land for $80,000, it will be based on the jurisdiction's statutory power of redemption. (A) is wrong because the deed of trust is a security interest (similar to a mortgage) to which the revocation rules for trusts do not apply. The deed of trust was created in part to allow the lender to foreclose on the property without going through a judicial foreclosure proceeding. (B) is wrong because, in states that permit a nonjudicial sale with deeds of trust containing a power of sale, the lender may bid at the sale, and in many cases the lender is the sole bidder. (D) is wrong because the prohibition against "clogging the equity of redemption" refers to the rule that a borrower's right to redeem his own mortgage cannot be waived in the instrument itself. Here, there is nothing to indicate that the owner's deed of trust prohibited him from redeeming the property prior to foreclosure. However, it is only through a statutory right of redemption that the owner would be able to redeem the property for $80,000 after the foreclosure sale had occurred.

A corporation was in the business of purchasing real property at below-market prices and reselling the properties to investors. The bylaws of the corporation authorized the chief executive officer ("CEO") and the director of the marketing division to enter into contracts on behalf of the corporation for the purchase or sale of properties. The corporation had recently purchased a large parcel of beachfront property for resale. The CEO secretly opened negotiations with an amusement park to sell the property. However, unknown to the CEO or anyone else in the corporation, the marketing director had already reached an agreement with a hotel for the sale of the property. On April 23, the marketing director and the hotel signed a written contract providing for sale of the property by the corporation to the hotel for $35 million. On April 25, the board of directors amended its bylaws, effectively depriving the marketing director of authorization to bind the corporation in purchase or sale transactions. This action was immediately publicized and became known to both the marketing director and the hotel. On April 26, the hotel duly recorded its contract. On May 1, the CEO, still unaware of the marketing director-hotel agreement, approved sale of the property to the amusement park for $39 million. The necessary documents of title were prepared and properly recorded by the amusement park on May 5. Two days later, the amusement park learned of the marketing director-hotel agreement. On May 10, the date scheduled for closing of the hotel's sale agreement, the CEO refused to accept the hotel's tender of $35 million and refused its demand for a deed to the property. The hotel subsequently brings action against the corporation and the amusement park for specific performance and to quiet title to the property. For whom will the court likely rule? (A) The defendants, because the board of directors had deprived the marketing director of authority to bind the corporation in the sale of real property. (B) The defendants, because the amusement park is the only purchaser who properly recorded a deed to the property. (C) The hotel, because the amusement park had constructive notice of the hotel's interests in the property when the agreement with the CEO was made. (D) The hotel, because the attempt to divest the marketing director of authority to approve sales of the corporation's property was invalid.

(C) Judgment should be for the hotel regardless of whether the jurisdiction has a notice statute or a race-notice statute. Under either type of recording statute, the only persons protected by the statute are bona fide purchasers. To attain this status, the person must take without notice—either actual, constructive, or inquiry—of the prior instrument. Because the marketing director-hotel contract was properly recorded, the amusement park had constructive notice of the hotel's interest in the property. Thus, the park could not become a bona fide purchaser when it entered into its contract. (A) is wrong because the marketing director had not been deprived of authority to bind the corporation at the time she signed the agreement with the hotel, and any subsequent change in her powers did not affect the validity of that agreement, nor the hotel's power to subsequently record the agreement. (B) is wrong because the hotel recorded its contract of sale. Any instrument creating or affecting an interest in land (e.g., deed, mortgage, contract to convey) can be recorded, providing constructive notice to subsequent purchasers. Thus, the hotel's failure to record a deed does not deprive it of protection of the recording statute. (D) is wrong because regardless of the validity of the board's attempt to divest the marketing director of authority, it is immaterial to the hotel's rights; it came after the valid marketing director-hotel contract was properly signed by the marketing director.

A cyclist was injured when a driver ran a red light. The cyclist subsequently sued the driver to recover for her injuries, and obtained a money judgment of $50,000. The state where the cyclist and the driver reside has the following statute: "Any judgment properly filed shall, for 10 years from filing, be a lien on the real property then owned or subsequently acquired by any person against whom the judgment is rendered." The cyclist filed the judgment in the county where the driver owned a valuable ranch. Sometime later, the driver, who was also injured in the accident, undertook to remodel all the buildings on the ranch to make them wheelchairaccessible. The driver borrowed $30,000 from a bank for the improvements, securing the loan with a mortgage on the ranch. The bank properly recorded its mortgage. Before he paid any principal on the bank's loan, the driver decided to build a new barn. He borrowed $20,000 from a financing company for this purpose, also secured by a mortgage on the ranch. The financing company properly recorded its mortgage. The driver subsequently defaulted on the bank's mortgage, and the bank brought a foreclosure action, joining the financing company in the proceeding. The foreclosure sale resulted in $90,000 in proceeds after all expenses and fees were paid. The driver still owes the cyclist $50,000, the bank $30,000, and the financing company $20,000. How should the foreclosure proceeds be distributed? (A) The cyclist is entitled to $50,000, the bank is entitled to $30,000, and the financing company is entitled to the remaining $10,000. (B) The cyclist is entitled to $50,000, the bank is entitled to $30,000, and the driver is entitled to the remaining $10,000. (C) The bank is entitled to $30,000, the financing company is entitled to $20,000, and the driver is entitled to the remaining $40,000. (D) The bank is entitled to $30,000, and the driver is entitled to the remaining $60,000.

(C) The bank is entitled to $30,000 of the foreclosure proceeds, the financing company is entitled to $20,000 of the proceeds, and the driver is entitled to the $40,000 balance. When an interest is foreclosed, after the expenses and fees are paid, the proceeds of the sale are first used to pay the principal and accrued interest on the loan that was foreclosed, next to pay off any junior liens, and finally any remaining proceeds are distributed to the mortgagor. Here, there are enough proceeds to satisfy the bank's (the foreclosing party's) $30,000 mortgage and the financing company's (the junior lienor's) $20,000 mortgage. The remaining balance ($40,000) is distributed to the driver (the mortgagor). (A) and (B) are wrong because the cyclist's interest, an interest senior to the bank's, is not affected by the foreclosure. Although foreclosure destroys all interests junior to the mortgage being foreclosed, it does not affect any senior interests. The buyer at the foreclosure sale takes subject to such interests. Without the cyclist foreclosing her lien, she is not entitled to a share of the proceeds, and her lien continues on the property in the buyer's hands. (B) and (D) are wrong because the financing company is entitled to have its mortgage fully discharged.

A landowner borrowed $30,000 from a bank, secured by a mortgage on his land. The mortgage papers were signed by the landowner and by the chief loan officer as agent for the bank on March 18. The loan officer filled out the appropriate recording form and gave it to a bank clerk on March 19, instructing him to file the papers at the county recorder's office. The bank clerk inadvertently misplaced the papers. He discovered the papers on April 10 and filed them with the county recorder. At the recorder's office, the bank clerk discovered a conveyance of the landowner's land from the landowner to a buyer dated April 5 and recorded on April 8. Subsequent inquiry revealed that the buyer paid the landowner $150,000 for the land after a diligent title search and that the buyer had no knowledge of the mortgage on the property until the loan officer contacted her on April 11. The jurisdiction in which the land is located follows the lien theory of mortgages, and has a statute providing: "Any conveyance of an interest in land shall not be valid against any subsequent purchaser for value without notice thereof who first records." If the bank seeks a declaration from the court that the buyer owns the land subject to a $30,000 mortgage with the bank, is the bank likely to prevail? (A) Yes, because the bank's interest was acquired for value prior to the date when the buyer recorded. (B) Yes, because the mortgage was merely security for a loan. (C) No, because the buyer recorded first. (D) No, because the jurisdiction follows the lien theory.

(C) The bank's position will not be upheld because the buyer, a bona fide purchaser, recorded first. The applicable recording statute is a race-notice statute, under which a subsequent purchaser for value without notice of any prior conveyance is protected if she records before the prior grantee. Here, the bank, as mortgagee, is a grantee of an interest in the land prior to the buyer. However, the buyer purchased the land for valuable consideration and without notice (either actual, record, or inquiry) of the prior conveyance to the bank. The buyer recorded her conveyance on April 8, prior to the time the bank clerk filed on behalf of the bank. Consequently, the buyer satisfies the statutory requirements, and she is protected against the bank's claim. (A) is incorrect because, Real Property Answers 9. to prevail, the bank must have actually recorded prior to the buyer. It is of no significance that the bank acquired its interest in the property—even for value—prior to the date of the buyer's recordation. The bank is not protected by the recording statute because it recorded its interest after recordation by another bona fide purchaser. (B) is incorrect because, although a mortgage is a security interest for a loan, it is still an instrument creating an interest in land. As such, a mortgage comes within the scope of the recording acts, so that a grantee thereof must record in order to give notice of the conveyance to subsequent purchasers. Having failed to record in time, the bank will lose against a subsequent bona fide purchaser who records first. (D) is incorrect because whether the jurisdiction is a lien theory or a title theory state is irrelevant to this question. Under either theory, the bank's mortgage is an instrument creating an interest in land, and is thus subject to the recording statute.

An entrepreneur opened a specialized business on her land. After using up most of her capital to purchase inventory, however, the entrepreneur needed more funds and asked her friend for a $30,000 loan, to be secured by the business's inventory. The friend declined the loan. A desperate entrepreneur then told the friend she would convey the land, which had a fair market value of $100,000, to him if he would give her the loan at the current market rate of interest. The friend agreed, and the entrepreneur conveyed the land to the friend the next day. At that time, the friend gave the entrepreneur $30,000 in cash, and the parties orally agreed that the entrepreneur would pay the friend back at the rate of $1,000 per month, and that after the loan was paid in full, the friend would reconvey the land to the entrepreneur. The friend immediately recorded his deed to the land. The entrepreneur made three $1,000 payments to the friend and then paid no more. She continued to live on the land but, being very much in debt, could not repay the loan. The friend, meanwhile, had received an offer to buy the land for $100,000. Which of the following most accurately states the friend's right to sell the property? (A) The friend may sell the land and keep the entire proceeds. (B) The friend may sell the land, but he must give $73,000 of the proceeds to the entrepreneur. (C) The friend may sell the land only after formally foreclosing on the property. (D) The friend may not sell the land.

(C) The friend may sell the land, but only after formally foreclosing on the property. If a deed is given for security purposes rather than as an outright transfer of the property, it will be treated as an "equitable" mortgage and the creditor will be required to foreclose it by judicial action like any other mortgage. In determining whether an absolute deed is really a mortgage, the court considers the following factors: (i) the existence of a debt or promise of payment by the deed's grantor; (ii) the grantee's promise to return the land if the debt is paid; (iii) the fact that the amount advanced to the grantor/debtor was much lower than the value of the property; (iv) the degree of the grantor's financial distress; and (v) the parties' prior negotiations. Here, the entrepreneur owed the friend a debt; the friend promised to return the property if the debt was paid; the amount advanced ($30,000) was much lower than the value of the property ($100,000); the entrepreneur was in great financial distress; and the parties' negotiations reveal that this transaction was intended as security for the loan. Thus, the friend must bring a judicial foreclosure proceeding before he can sell the land. (A) is wrong because a foreclosure is required. Furthermore, even in a foreclosure sale, the friend is not entitled to all of the proceeds. The proceeds are used to first pay the expenses of the sale, attorneys' fees, and court costs; then to pay the principal and accrued interest on the loan that was foreclosed; then to pay off junior interests. Any remaining proceeds are returned to the mortgagor. The friend is entitled only to his expenses and the amount still 10. Real Property Answers owing on the $30,000 loan, including accrued interest. Because the friend has a buyer willing to pay $100,000, the entrepreneur should get some money back. (B) is wrong for two reasons: (i) as explained above, the friend cannot sell the property without a judicial foreclosure; and (ii) the entrepreneur would not be entitled to $73,000. The friend is entitled to his expenses of sale and the principal amount owing, plus accrued interest. (D) is wrong because the friend can sell the land, provided he undertakes formal foreclosure proceedings.

An uncle validly executed and notarized a deed conveying his beach house to his nephew, and then validly recorded the deed. When the nephew, who was experiencing financial difficulties, learned of the recordation of the deed, he immediately told his uncle that he did not want the beach house and could not accept such an expensive gift anyway. Later, the nephew filed for bankruptcy and the trustee in bankruptcy asserted an ownership interest in the beach house on behalf of the debtor's estate. The bankruptcy court ruled that the property belonged to the uncle and not to the nephew, and thus was not part of the debtor's estate subject to distribution. Which of the following is the strongest reason in support of the bankruptcy court's ruling? (A) There was no presumption of delivery created by recordation of the deed because the nephew did not know of the recordation. (B) The nephew's statements to the uncle were a constructive reconveyance of the property. (C) There was never an effective acceptance of delivery of the deed by the nephew. (D) The recordation of the deed was invalid because it was done without the nephew's permission.

(C) The nephew's express rejection of the deed was sufficient to rebut any presumption of acceptance. As a general rule, delivery of the deed is the final operative act to complete a conveyance of title to the grantee, because courts will infer the grantee's acceptance if the conveyance is beneficial to him. However, all courts will consider evidence that is contrary to the presumption or inference. Hence, the nephew's express rejection of the gift is sufficient to establish that no conveyance of the property took place. (A) is an incorrect statement of law. If the grantor intends the recording of the deed to be the final act in vesting title in the grantee, then such recording creates a presumption of delivery even where the grantee did not know of the recordation. (B) is wrong because there is no such thing as a constructive reconveyance. Had the nephew accepted the gift (completing the conveyance) and later changed his mind, the nephew would have had to execute a new deed to convey the property back to the uncle. (D) is wrong because knowledge or permission of the grantee has no effect on the validity of the recordation; rather, it determines whether there has been an effective acceptance.

A woman purchased a tract of land from a man by warranty deed. Unbeknownst to the woman, the man was not the actual owner of the tract. The woman built a home on the tract and moved into it. Two years later, the actual owner learned of the man's transaction with the woman and prevented the woman from entering the tract from that point forward. This led to a costly court battle. When the woman notified the man and told him that she thought it was his duty to straighten this out, he ignored her. The statute of limitations for actions on deed covenants is four years. The woman would succeed in a suit for damages against the man for breach of which of the following covenants of title? (A) The covenant of quiet enjoyment only. (B) The covenants of seisin, right to convey, quiet enjoyment, warranty, further assurances, and the covenant against encumbrances. (C) The covenants of seisin, right to convey, quiet enjoyment, warranty, and further assurances. (D) The covenants of seisin and right to convey only.

(C) The woman would succeed in a suit for damages against the man for breach of the covenants of seisin, right to convey, quiet enjoyment, warranty, and further assurances, but not on the covenant against encumbrances. A general warranty deed gives the grantee six covenants of title: the right to seisin, the right to convey, a covenant against encumbrances, the covenant of quiet enjoyment, the covenant of further assurances, and a general warranty. Under the covenants of quiet enjoyment, warranty, and further assurances, the man promised that (i) the woman would not be disturbed in her possession of the tract; (ii) he would defend the woman's title against lawful claims; and (iii) he would perform whatever acts are necessary to perfect the woman's title. Because the man neither owned the tract of land nor was acting as the actual owner's agent, he breached the covenants of seisin and right to convey at the time of the conveyance to the woman. When the actual owner prevented the woman from re-entering the property, this interfered with the woman's quiet enjoyment, and the man's refusal to "straighten this out" was a breach of the covenant of further assurances. Thus, (C) is the correct answer. Hence, (A) is incorrect because quiet enjoyment was not the only covenant breached. There is nothing in the facts to suggest the property is encumbered; thus, the man did not breach the covenant against encumbrances, and (B) is therefore incorrect. (D) is incorrect because seisin and right to convey were not the only covenants that the man breached.

A seller put her house and lot on the market for $200,000. After receiving several offers within $5,000 of her asking price, the seller entered into a contract to sell the house and lot to a buyer for $200,000. The contract provided that the buyer put up $4,000 in earnest money, which the seller could treat as liquidated damages unless: The seller fails to tender marketable title to the buyer by the agreed-upon closing date, the seller commits a material breach of this contract, or the buyer dies prior to the closing date, in which case the earnest money shall be reimbursed to the buyer's estate. The contract was signed on July 24, and the closing date was set for September 12. On August 5, the buyer was seriously injured in an accident. On September 10, the buyer was released from the hospital in a wheelchair. He determined that a ranch-style house would make his life much more bearable, but the seller's home was two stories. The buyer asked the seller to cancel the contract and to refund the $4,000 earnest money. The seller refused. The buyer did not appear on the closing date. On September 16, the seller contracted to sell the home to a purchaser for $198,000. The closing occurred as planned on October 20. The buyer files suit against the seller, praying for a refund of the $4,000 earnest money. How much is the buyer likely to recover? (A) The entire $4,000, because the buyer had a justified medical reason for his failure to perform. (B) $2,000, because the diminution in value of the property was only $2,000. (C) $2,000 less any of the seller's out-of-pocket costs involved in remarketing the home. (D) Nothing, because at the time the contract was entered into, $4,000 represented a reasonable estimate of damages in the event of breach.

(D) The buyer will most likely recover nothing because, at the time of the contract, $4,000 represented a reasonable estimate of damages in the event of breach. When a sales contract provides that a seller may retain the buyer's earnest money as liquidated damages, courts routinely uphold the seller's retention of the money upon breach if the amount appears reasonable in light of the seller's anticipated and actual damages. Many courts uphold retention of earnest money of up to 10% of the sales price without inquiry into its reasonableness. In this case, the earnest money represented 2% of the purchase price. Given the fact that the seller had received other offers within $5,000 of the price offered by the buyer, $4,000 would be a reasonable estimate of damages if the seller were forced to accept another offer. (A) is wrong because the fact that the buyer had a good reason for not performing does not change the fact that he is in breach. The contract is not impossible for the buyer to perform; it is just not as attractive a purchase as it was before the accident. He cannot escape liability on this basis. (B) and (C) are wrong because if there is a valid liquidated damages clause, it will be enforced and actual damages are irrelevant. (If the liquidated damages clause were not enforceable, (C) would be a better choice than (B) because the seller would be entitled to her expenses in remarketing the property.) Thus, because the liquidated damages clause is enforceable, the buyer will not be able to recover any of the $4,000 he paid as earnest money.

A landowner embarked on an expedition into a remote jungle, leaving no means to communicate with him. Because property values suddenly began plummeting in the landowner's neighborhood, his son believed that it was imperative to sell his father's property before it became worthless. Having no way to speak to his father ahead of time, the son prepared a deed conveying the property to a buyer, but left the line for the buyer's name blank. He then signed his father's name on it as the grantor, and handed the deed to the buyer. The deed, however, did not include any language regarding the amount the father was to receive in exchange for the property. The buyer believed that the son was the owner of the property. When the father returned, he was happy that the property had been sold. If the buyer changed his mind and now wishes to have the conveyance set aside, which of the following would be his best argument? (A) The deed was not valid because the rapidly declining property values amounted to extreme duress. (B) The deed was not valid because the buyer was not identified in the writing. (C) The deed was not valid because the consideration for the deed was not contained in the writing. (D) The deed was not valid because the son signed it.

(D) The buyer's best argument would be that the deed is void because the son signed it. A valid deed requires a writing containing a description of the land and parties, words of intent, and the grantor's signature. Here, the signature on the deed was forged. A defective deed may be voidable, which means that it would be set aside only if the property had not been conveyed to a bona fide purchaser, or it could be void, meaning that the deed would be set aside regardless of the property having passed to a bona fide purchaser. Deeds obtained by means of, among other things, duress, Real Property Answers 5. undue influence, or mistake are considered voidable. Deeds that were forged, never delivered, or obtained by fraud in the factum are void. Here, although the father seems to have ratified the conveyance and the buyer was a bona fide purchaser, the deed is void because the signature was forged. (A) is incorrect. A deed obtained by duress would be merely voidable, and in any case, the pressure caused by rapidly dropping values is not the duress that is contemplated by the common law rule. Duress for voidability purposes means pressure that is brought by an individual or entity in order to procure the deed. (B) is incorrect. Although a deed must identify the parties, courts will presume that the person taking delivery is authorized to fill in the name of the grantee. In the absence of the son's forgery—i.e., if the landowner had executed and handed the deed to the buyer—all the buyer would have had to do was fill in his name and the deed would be valid. (C) is incorrect because a recitation of the consideration for the conveyance is not required to make a deed valid.

A man gave a friend a deed purporting to give to the friend "My property known as Twelve Oaks, with its five acres of land and the stable and dressage course located thereon." The man told the friend that he was giving the property to her, but because he did not have a better description, he wanted to keep the deed until his attorney could review it. The friend agreed and gave it back to him. Unfortunately, the man suffered a heart attack the next day and died without seeing his attorney. However, when the friend spoke with the administrator of the man's estate, she learned that the man had, as part of another deed 12 years before, sold that part of Twelve Oaks on which was located the stable and dressage course. The man's heirs bring an action for declaratory relief against the friend, asserting that the deed to Twelve Oaks is void because it contained an inaccurate and ambiguous description. How will the trial court rule? (A) Against the friend, because the deed failed to give an accurate description of the property owned by the man at the time of his death. (B) Against the friend, because the man informed the friend that he did not know the description of his property and he wanted his attorney to prepare the deed. (C) Against the friend, because the deed purports to transfer more property than the man owned and thus is void as a matter of law. (D) For the friend, because the error in description was not sufficient to put in doubt what the man intended to convey to her.

(D) The court should find for the friend, because the error in description was not sufficient to put in doubt what the man intended to convey to her. In land contracts and deeds, property may be described in various ways; i.e., by reference to a government survey, by metes and bounds, by courses and angles, by references to a recorded plat, by reference to adjacent properties, by the name of the property, or by a street and number system. A description in a deed is sufficient if it furnishes a good lead as to the identity of the property. Hence, (A) is wrong. (C) is incorrect because a deed purporting to convey m

A landowner owned a large piece of property containing an inn and a bakery. She entered into a contract to sell the property to a purchaser for $1 million. The contract was recorded. The purchaser gave the landowner $200,000 as earnest money. The closing date was set for September 10, two months after the signing of the contract. On August 10, an arsonist set fire to the inn, which burned to the ground. On September 10, the landowner appeared at the closing and tendered the deed to the property. The buyer refused to tender the remaining $800,000 of the purchase price and demanded the return of his earnest money. The landowner sued the buyer for specific performance of the contract. The buyer countersued for the return of his earnest money. Both parties stipulate that the value of the property without the inn is $600,000, that insurance on the property had lapsed, and that the common law, unmodified by statute, applies. What is the most likely result at trial? (A) The landowner will not prevail on the issue of specific performance, but will be allowed to keep the earnest money. (B) The landowner will not prevail on the issue of specific performance and will be ordered to return the earnest money. (C) The landowner will prevail on the issue of specific performance, but the price will be abated to $600,000. (D) The landowner will prevail on the issue of specific performance for the full contract price.

(D) The landowner will succeed in her suit for specific performance at the full contract price. Where property subject to a contract for sale is destroyed without the fault of either party before the date set for closing, the rule in the absence of a statute is that the risk of loss is on the buyer. Thus, the buyer must pay the contract price despite a loss due to fire, unless the contract provides otherwise. Here, the inn was destroyed by fire after the landowner and the buyer entered into their contract for the sale of the property, but before the closing date. The contract apparently was silent regarding the risk of loss and there is no applicable statute. Thus, under the common law rule, the risk of loss is on the buyer. As a result, the landowner is entitled to receive specific performance of the contract, meaning that the buyer must pay the full contract price. (A) and (B) are incorrect because they conclude that the landowner is not entitled to specific performance. As explained above, the landowner is entitled to specific performance because the risk of loss is on the buyer. (B) is also incorrect because it states that the landowner must refund the earnest money. The landowner is entitled to the full contract price; thus, there is no reason for her to return the earnest money. (C) is incorrect because it allows the buyer to tender less than the full contract price. With the buyer bearing the risk of loss, he must pay the $1 million contract price despite the decrease in the property's value due to the fire.

A rancher entered into a written contract to buy a farm from a farmer for $100,000. The contract stipulated for closing on September 30. In addition, the contract contained the following provision: "The taxes shall be prorated as agreed to by the parties at a later date." Upon the signing of the contract, the rancher gave the farmer a check for $10,000 as a down payment. On September 28, the rancher notified the farmer that he would not be able to close on the farm until October 2, because the closing on his current home, the proceeds from which were to be applied to his purchase of the farm, was unavoidably delayed due to his buyer's illness. Meanwhile, the farmer had difficulty finding a home she liked as well as the farm. She decided that she would rather not sell the farm and wished to avoid the contract with the rancher. On October 2, the rancher showed up at the closing with the $90,000 to tender to the farmer. The farmer did not show up. The rancher sues for specific performance. In whose favor will the court most likely rule? (A) The farmer, because the tax provision is an essential term of the contract, and it is not specific enough to satisfy the Statute of Frauds. (B) The farmer, because the rancher materially breached by not tendering performance on September 30. (C) The rancher, because of the operation of the doctrine of equitable conversion. (D) The rancher, because time was not of the essence.

(D) The rancher will prevail because there is no evidence that time was of the essence. In general, courts presume that time is not of the essence in real estate contracts. Thus, the closing date stated in the contract is not absolutely binding in equity, and a party, even though late in tendering his own performance, can still enforce the contract if he tenders within a reasonable time. (One to two months is usually considered reasonable.) Time will be considered of the essence only if: (i) the contract so states, (ii) the circumstances indicate it was the parties' intention, or (iii) one party gives the other notice that he desires to make time of the essence. The contract in this case made no mention that time was of the essence. The facts do not indicate any circumstances, such as rapidly fluctuating prices or the need for the money to close another critical transaction, that would indicate that the rancher and the farmer intended time to be of the essence. The farmer did not give the rancher reasonable notice before September 30 that she wanted to make time of the essence. Thus, the court will not find that time is of the essence here. Because time is not of the essence, the rancher is not in material breach and is entitled to specific performance. (A) is wrong because the Statute of Frauds is not violated here. Contracts for the sale of land must be in writing to be enforceable. The essential terms for purposes of the Statute of Frauds are: the description of the property, the identification of the parties, and the price. The tax provision is not an essential term. It is an incidental matter, which need not appear in writing or even be agreed upon. (B) is wrong because, as discussed above, the rancher is not in material breach. Time was not of the essence, so the fact that the rancher did not tender his performance on September 30 did not constitute a breach of the land sale contract. (C) is wrong because the doctrine of equitable conversion will not affect the rights of the parties in this situation. The doctrine of equitable conversion holds that once an enforceable contract of sale is signed, the purchaser's interest is real property, and the seller's interest (the right to proceeds) is personal property. This is important with respect to which party bears the risk of loss if the property is damaged before the date set for closing or if one of the parties dies prior to closing. It has no effect in situations like this one where the question in issue is the enforceability of the contract itself.

A landowner owned a large tract of land containing numerous coal mines. To finance the renovation of some of the buildings on the land, the landowner obtained a $50,000 mortgage from a bank. Shortly thereafter, the landowner, without notifying anyone of the bank's interest, sold the surface of the land to his sister and the mineral rights to a utility company. The bank recorded its mortgage the next day; the day after that, the utility company recorded its deed; the following day, the sister recorded her deed. None of the parties dealing with the landowner had any knowledge of the others at the time of their transactions. The jurisdiction in which the land is located has the following statute: "No conveyance or mortgage of an interest in land is valid against any subsequent purchaser for value without notice thereof whose conveyance is first recorded." If the sister brings an action to quiet title to the land, what would be the most likely result? (A) The sister would have only a reversionary interest. (B) The sister would have a fee simple absolute free of the interests of the bank and the utility company. (C) The sister would have a fee simple absolute subject only to the payment of the mortgage held by the bank. (D) The sister would have a fee simple interest subject to the mineral rights of the utility company and the mortgage held by the bank.

(D) The sister's fee simple ownership of the land would be subject to the bank's mortgage interest and the utility company's mineral interest. Under a race-notice statute, which the jurisdiction in this question has, a subsequent bona fide purchaser (i.e., one who takes for value and without notice) is protected only if she records before the prior grantee. Notice is measured at the time of the conveyance, not at the time of recording. The rationale of this type of statute is that the best evidence of which interest was created first is to determine who recorded first. As an inducement to record promptly, race-notice statutes impose on the bona fide purchaser the additional requirement that she record first. Because the bank was the first to receive a conveyance, the bank could not be held to have knowledge of any other conveyance, and when the bank recorded its conveyance first, the bank won out over the sister and the utility company under the statute. The utility company can enforce its mineral interest in coal on the land because it recorded before the sister. (A) is incorrect because the sister has a present ownership interest in the land, but it is subject to the bank's mortgage and the utility company's mineral interest. (B) is incorrect because the sister takes subject to the bank's and utility's interests. As noted above, a subsequent bona fide purchaser is protected under a race-notice statute only if she records before a prior grantee. Although the sister took without notice, she was the last to record, and thus the bank's and utility's interests prevail. (C) is incorrect because, as discussed above, the sister does not have a fee simple absolute; the utility company owns the mineral interest.

A large tract of land was located in a jurisdiction that has adopted the following statute: No conveyance or mortgage of an interest in land is valid against a subsequent purchaser for value without notice thereof whose conveyance is first recorded. The man who owned the land owed money to a woman, and in satisfaction of this debt, the man conveyed the property to her. Although the woman intended to have the deed recorded, she mistakenly failed to do so. Two years later, the man borrowed money from a bank and, to secure the loan, executed a mortgage deed on the property. The bank promptly recorded this mortgage. Three months later, the man, just before he died, donated the property by general warranty to his son, who did not know about the prior events. The son recorded the deed and entered into a contract with his friend to sell him the property. The next month, the woman discovered that the deed in her safe was not recorded, and so, without notice of any of the prior transactions, the woman recorded the deed. A month after that, the friend paid the son full value for the property, and without actual knowledge of any of the other transactions regarding the property, the friend had the deed duly recorded. By the end of the next year, the friend had expended substantial sums of money on the property. However, when he put up the property as security for a loan from the bank, he learned for the first time of the woman's claim. In a suit between the friend and the woman, which of the following statements most accurately describes the probable outcome? (A) The friend would prevail, because the money he paid for the property, along with the money expended since then, was far in excess of what the woman paid, and under equity, the friend would be deemed the owner; however, he would have to reimburse the woman for what she paid for the property. (B) The friend would prevail, because under the doctrine of equitable conversion, his "right" to the property preceded the woman's recordation, and thus whatever right she may have had would have been terminated before she could record. (C) The friend would prevail, because he purchased from the son, whose deed was recorded before the woman's deed. (D) The woman would prevail, because she recorded first.

(D) The woman would prevail because she recorded her deed before the friend recorded his deed. The jurisdiction in this question has a race-notice statute, under which a subsequent bona fide purchaser is protected only if she records before the prior grantee. While the friend was a bona fide purchaser, he did not record his interest in the property before the woman did; thus the woman will prevail. (A) is incorrect because priority under a race-notice recording act, such as the one in this question, is determined by the subsequent purchaser's status as a bona fide purchaser and on the basis of who records first. Courts do not determine ownership by balancing the equities on the basis of who spent the most money. (B) is wrong because the doctrine of equitable conversion, wherein equity regards the purchaser in a land sale contract as the owner of the real property, does not change the result under the recording statute. The woman will prevail because she recorded first. (C) is incorrect because the friend cannot rely on the son's recording of his deed; the woman recorded her interest before the friend recorded his. Furthermore, even if the issue were whether the friend had actual notice of the woman's interest, the friend could not rely on the "shelter rule" that protects transferees from a bona fide purchaser, because the son was not a purchaser for value and therefore not protected by the recording statute.

Which of the following leases does not create a month-to-month periodic tenancy? A"L leases to T at an annual rent of $6,000, payable at $500 per month" B"L leases to T from month to month" C"L leases to T at a rent of $500 per month" DL orally leases to T "for the next few years" and accepts monthly rent payments

A "L leases to T at an annual rent of $6,000, payable at $500 per month" does not create a month-to-month periodic tenancy. A periodic tenancy is a tenancy that continues from period to period until terminated by proper notice by either the landlord or the tenant. It may be created in three ways: (i) by express agreement; (ii) by implication if a lease with no set termination date provides for the payment of periodic rent; or (iii) by operation of law if the tenant holds over or pays rent periodically under an invalid lease. Although a lease at an annual rent, payable monthly creates a periodic tenancy, the majority view is that the tenancy is from year to year rather than month to month. "L leases to T from month to month" creates a month-to-month periodic tenancy by express agreement. "L leases to T at a rent of $500 per month" creates a month-to-month periodic tenancy by implication because the lease reserves a monthly rent. If L orally leases to T "for the next few years" and accepts monthly rent payments, the lease is invalid because most states require that a lease creating a tenancy for more than one year be in writing to satisfy the Statute of Frauds. However, the tenant's monthly payment of rent converts what would otherwise be a tenancy at will, terminable at the will of either the landlord or the tenant, into a month-to-month periodic tenancy.

A co-tenant has the right to possess: AThe entire estate BHis proportion of the estate, as determined by his amount of consideration paid CHis proportion of the estate, as determined by the number of co-tenants

A A co-tenant has the right to possess the entire estate subject to the equal right of his co-tenants. A co-tenant out of possession cannot bring a possessory action unless there has been an "ouster" (i.e., wrongful exclusion) by the co-tenant in possession. Although tenants in common may own unequal interests in the property (e.g., A owns 2/3, B owns 1/6, and C owns 1/6), they have the equal right to possess the entire estate. A co-tenant's right to possession is not his proportion of the estate as determined by his amount of consideration paid or the number of co-tenants. Note that joint tenants may pay unequal amounts of consideration, but they always hold equal interests in the property (i.e., unity of interest)

A fee simple subject to an executory interest is an estate that: AAutomatically divests in favor of a third party on the happening of a stated event BContinues after the happening of a stated event until the grantor exercises her power of termination CContinues after the happening of a stated event until the third party exercises his power of termination DAutomatically terminates on the happening of a stated event and reverts to the grantor

A A fee simple subject to an executory interest is an estate that automatically divests in favor of a third party (rather than the grantor) on the happening of a stated event. It is created by the same language used to create a fee simple determinable (e.g., "for so long as," "while," "during," or "until") or a fee simple subject to a condition subsequent (e.g., "upon condition that," "provided that," "but if," or "if it happens that"), but rather than automatically reverting to the grantor on the happening of a stated event (fee simple determinable) or continuing after the happening of a stated event until the grantor exercises her power of termination (fee simple subject to a condition subsequent), it automatically divests in favor of a third party on the happening of a stated event. A fee simple subject to an executory interest is not an estate that continues after the happening of a stated event until the third party exercises his power of termination. An estate that continues on the happening of a stated event until the grantor exercises her power of termination (right of entry) is a fee simple subject to a condition subsequent. A right of entry can be created only in favor of the grantor and her heirs. If a similar interest is created in favor of a third party, it is called an executory interest. However, unlike a right of entry, the third party need not "exercise" his executory interest; on the happening of the stated event, the estate automatically divests in his favor. The common law did not recognize a future interest created in a third party that would vest in possession only upon the discretionary exercise of a right or power by the third party. A fee simple subject to an executory interest is not an estate that automatically terminates on the happening of a stated event and reverts to the grantor. As explained above, such an estate is a fee simple determinable. A fee simple subject to an executory interest is not an estate that continues after the happening of a stated event until the grantor exercises her power of termination. As explained above, such an estate is a fee simple subject to a condition subsequent.

A grantor who conveys a fee simple absolute retains: ANo interest BA possibility of reverter CA reversion

A A grantor who conveys a fee simple absolute retains no interest. A fee simple absolute is the largest estate in property permitted by law. It has an indefinite and potentially infinite duration. The holder can sell it, divide it, or devise it, and it passes to his heirs. If O owned a fee simple absolute, a conveyance from "O to A" creates a fee simple absolute in A, leaving O with no interest. A grantor who conveys a fee simple determinable retains a possibility of reverter. A fee simple determinable is an estate of potentially infinite duration, but which automatically terminates on the happening of a stated event and reverts to the grantor. A conveyance from "O to A for so long as/while/during/until [event]" creates a fee simple determinable in A and a possibility of reverter in O. A grantor who conveys a lesser estate than the grantor owns retains a reversion. Where O has a fee simple, a conveyance from "O to A for life" creates a life estate in A and a reversion in O.

A grantor who conveys a fee simple determinable retains __________. AA possibility of reverter BA reversion CA right of entry DNo interest

A A grantor who conveys a fee simple determinable retains a possibility of reverter. A conveyance from "O to A for so long as/while/during/until [event]" creates a fee simple determinable in A and a possibility of reverter in O. A right of entry is the future interest retained by a grantor who conveys a fee simple subject to a condition subsequent. A conveyance from "O to A upon condition that/provided that/but if/if it happens that [event], then O or her heirs may enter and terminate the estate" creates a fee simple subject to a condition subsequent in A and a right of entry in O. A reversion is the future interest retained by a grantor who conveys a lesser estate than the grantor owns. Where O has a fee simple, a conveyance from "O to A for life" creates a life estate in A and a reversion in O.

Which of the following is not a future interest in the grantor? ARemainder BRight of entry CReversion DPossibility of reverter

A A remainder is a future interest created in a grantee rather than a grantor. A remainder must be expressly created in the instrument creating the intermediate possessory estate. A conveyance from "O to A for life, then to B" creates a life estate in A and a remainder in B. A possibility of reverter is the future interest left in a grantor who conveys a fee simple determinable. A conveyance from "O to A for so long as/while/during/until [event]" creates a fee simple determinable in A and a possibility of reverter in O. A right of entry is the future interest retained by the grantor who conveys a fee simple subject to a condition subsequent. A conveyance from "O to A upon condition that/provided that/but if/if it happens that [event], then O or her heirs may enter and terminate the estate" creates a fee simple subject to a condition subsequent in A and a right of entry in O. A reversion is the future interest left in a grantor who conveys a lesser estate. A conveyance from "O to A for life" creates a life estate in A and a reversion in O. A. PRESENT POSSESSORY ESTATESA present possessory estate is an interest that gives the holder the right to present possession. 1. Fee Simple AbsoluteA fee simple absolute is the largest estate recognized by law. It can be sold, divided, devised, or inherited and has an indefinite or potentially infinite duration. Today, a fee simple is presumed in the absence of express contrary intent (words of inheritance are no longer necessary).2. Defeasible FeesDefeasible fees are fee simple estates (i.e., of uncertain or potentially infinite duration) that can be terminated upon the happening of a stated event. a. Fee Simple Determinable (and Possibility of Reverter)A fee simple determinable terminates upon the happening of a stated event and automatically reverts to the grantor. It is created by durational language, such as "for so long as," "while," "during," or "until." A fee simple determinable can be conveyed, but the grantee takes subject to the estate's being terminated by the specified event.EXAM TIPRemember that statements of motive or purpose do not create a determinable fee. To create a fee simple determinable, words limiting the duration of the estate must be used. Watch for grants such as "for the purpose of" and "to be used for"); they are merely expressions of motive. 1) Correlative Future Interest in Grantor - Possibility of ReverterWhenever a grantor conveys a fee simple determinable, he automatically retains a possibility of reverter, which is a reversionary future interest. A possibility of reverter is transferable, descendible, and devisable. b. Fee Simple Subject to Condition Subsequent (and Right of Entry)A fee simple subject to a condition subsequent is an estate in which the grantor reserves the right to terminate the estate upon the happening of a stated event); i.e., the estate does not automatically terminate - the grantor must take some action. The estate is created by use of conditional words, such as "upon condition that," "provided that," "but if," and "if it happens that." 1) Correlative Future Interest in Grantor - Right of EntryThe right to terminate, reserved by the grantor, is called a right of entry. It must be expressly reserved); in contrast with a possibility of reverter, it does not arise automatically. Most courts hold that rights of entry are not transferable inter vivos, but most states agree they are devisable, and all states agree they are descendible. EXAM TIPA conveyance that contains both durational language and a power of termination will likely be construed as creating a fee simple subject to a condition subsequent, because the forfeiture is optional at the grantor's election rather than automatic. Policy disfavors forfeiture of estates.c. Fee Simple Subject to an Executory InterestIf a fee simple estate terminates upon the happening of a stated event (because it is determinable or subject to a condition subsequent) and then passes to a third party rather than reverting to the grantor or giving the grantor a right to terminate, the third party has an executory interest.Examples: 1) "To A and his heirs for so long as liquor is not sold on the premises); in that event, to B." B has an executory interest.2) "Blackacre to XYZ Church, but if it is used for anything other than church purposes, then to B." B has an executory interest.d. Conditions and Limitations Violating Public PolicyConditions or limitations that violate public policy generally are struck down, and the grantee takes free of the restraint. If the purpose of the condition is to penalize marriage or encourage divorce, it likely will be struck down. However, if the purpose is to give support until marriage or in the event of divorce, it likely will be upheld. 3. Fee TailThe fee tail is an estate where inheritability is limited to lineal heirs. It is created by the words "to A and the heirs of his body." Most jurisdictions have abolished the fee tail, and an attempt to create one results in a fee simple.4. Life EstateA life estate is one measured by the life or lives of one or more persons. It may be created by operation of law (e.g., dower) or by conveyance. a. Life Estates by Marital Right (Legal Life Estates)Dower and curtesy were the common law interests of a spouse in the real property of the other spouse. These interests could not be defeated by conveyance or by creditors. Most states have abolished dower and curtesy in favor of a statutory right to a portion of a spouse's estate.b. Conventional Life Estates 1) For Life of GranteeThe usual life estate is measured by the life of the grantee (e.g., "to A for life"). This type of life estate may be implied from language such as "to B after the life of A."2) Life Estate Pur Autre Vie (Life of Another)A life estate "pur autre vie" is measured by a life other than the grantee's (e.g., "to A for the life of B"). A life estate pur autre vie also results when the life tenant conveys his life estate to another (e.g., if A, the holder of a life estate, conveys his interest to B, B has a life estate for the life of A).EXAM TIPAlthough a life estate is usually indefeasible (i.e., it ends only when the life tenant dies), it is possible to create life estates that are defeasible in the same ways that fee estates can be defeasible. A life estate can be determinable, subject to a condition subsequent, and subject to an executory interest (e.g., "to A for life so long as alcohol is not used on the premises" or "to A for life, but if A is divorced, to B"). CHART c. Rights and Duties of Life Tenant - Doctrine of WasteA life tenant is entitled to any ordinary uses and profits of the land, but cannot do anything that injures the interests of a remainderman or reversioner. A future interest holder may sue for damages or to enjoin such acts, and if she spends money to perform the life tenant's obligations (see 2, infra), she is entitled to reimbursement. 1) Affirmative (Voluntary) Waste - Natural ResourcesExploitation of natural resources (e.g., minerals) by a life tenant is generally limited to situations when: (i) necessary for repair or maintenance of the land); (ii) the land is suitable only for such use); or (iii) it is expressly or impliedly permitted by the grantor. Under the open mines doctrine, if mining was done on the land prior to the life estate, the life tenant can continue mining - but is limited to the mines already open.2) Permissive WasteA life tenant is obligated to: (i) preserve the land and structures in a reasonable state of repair); (ii) pay interest on mortgages (not principal)); (iii) pay ordinary taxes on the land); and (iv) pay special assessments for public improvements of short duration (improvements of long duration are apportioned between the life tenant and future interest holder). Permissive waste occurs when a life tenant fails to do so. However, this duty is limited to the extent of the income or profits generated from the land (or to its reasonable rental value, if there is no income or profit). A life tenant is not obliged to insure the premises for the benefit of remaindermen and is not responsible for damages caused by a third-party tortfeasor.3) Ameliorative WasteAmeliorative waste is a change that benefits the property economically. This waste was actionable at common law, but now a life tenant may alter or even demolish existing buildings if:(i) The market value of the future interests is not diminished); and either(ii) The remaindermen do not object); or(iii) A substantial and permanent change in the neighborhood conditions (e.g., change from residential to 90% industrial) has deprived the property in its current form of reasonable productivity or usefulness. a) Compare - Leasehold TenantLeasehold tenants remain liable for ameliorative waste even if the neighborhood has changed and the market value of the premises was increased.b) Compare - Worthless PropertyIf the land is practically worthless in its present state, the life tenant may seek a partition sale, the proceeds of which are put in trust with income paid to the life tenant. d. Renunciation of Life EstateIf a life tenant who receives the estate by will or intestacy renounces his interest, the future interest following the life estate is generally accelerated so that it becomes immediately possessory. 5. Estate for Years, Periodic Estate, Estate at Will, Tenancy at SufferanceThese present estates are considered in the next chapter, which concerns the landlord-tenant relationship. B. FUTURE INTERESTSA future interest gives its holder the right or possibility of future possession of an estate. It is a present, legally protected right in property. 1. Reversionary Interests - Future Interests in Transferor a. Possibilities of Reverter and Rights of EntryThese interests are discussed supra in connection with defeasible fees. CHART b. ReversionsA reversion is the estate left in a grantor who conveys less than she owns (e.g., O conveys "to A for life"); O has a reversion). It arises by operation of law); it does not have to be expressly reserved. A reversion is alienable, devisable, and inheritable. Its holder can sue for waste and for tortious damage to the reversionary interest. EXAM TIPAll reversionary interests are vested and, thus, not subject to the Rule Against Perpetuities.2. RemaindersA remainder is a future interest in a third person that can become possessory on the natural expiration of the preceding estate. It cannot divest a prior estate, and it cannot follow a time gap after the preceding estate. A remainder must be expressly created in the instrument creating the preceding possessory estate.Examples: 1) O conveys "to A for life, then to B and his heirs"); B has a remainder.2) O conveys "to A for life, then to B and his heirs one day after A's death "); B does not have a remainder (because there is a gap).EXAM TIPBecause a remainder cannot "cut short" a preceding estate, it can never follow a fee simple estate, which is of potentially infinite duration. Executory interests are the future interests that cut short preceding estates or follow a gap after them. a. Indefeasibly Vested RemainderA vested remainder is one created in an existing and ascertained person, and not subject to a condition precedent. The remainderman has a right to immediate possession upon normal termination of the preceding estate. An indefeasibly vested remainder is a vested remainder that is not subject to divestment or diminution.b. Vested Remainder Subject to OpenThis is a vested remainder created in a class of persons (e.g., "children") that is certain to become possessory, but is subject to diminution - e.g., by the birth of additional persons who will share in the remainder as a class.Example: O conveys "to A for life, then to the children of B." A and B are living and B has one child, C. C has a vested remainder subject to open.c. Vested Remainder Subject to Total DivestmentThis is a vested remainder that is subject to a condition subsequent.Example: O conveys "to A for life, then to B and his heirs); but if B dies unmarried, then to C and his heirs." B has a vested remainder subject to complete divestment by C's executory interest.EXAM TIPWhere language is ambiguous, the preference is for vested remainders subject to divestment rather than contingent remainders or executory interests. Policy favors early vesting of estates.d. Contingent RemainderContingent remainders are those created in unborn or unascertained persons, or subject to a condition precedent. 1) Subject to Condition PrecedentA condition is precedent if it must be satisfied before the remainderman has a right to possession.Examples: 1) O conveys "to A for life, then to B and his heirs if B marries C." B's remainder is contingent because he must marry C before he can take possession.2) O conveys "to A for life, then to B and his heirs if B marries C, otherwise to D and his heirs." B and D have alternative contingent remainders.Compare: O conveys "to A for life, then to B and his heirs); but if B marries C, then to D and his heirs." B has a vested remainder (because no condition precedent) subject to divestment by D's executory interest.2) Unborn or Unascertained PersonsA remainder created in unborn or unascertained persons is contingent because until the remainderman is ascertained, no one is ready to take possession if the preceding estate ends.Example: O conveys "to A for life, then to the children of B." If B is childless at the time, the remainder is contingent.3) Destructibility of Contingent RemaindersAt common law, a contingent remainder was destroyed if it failed to vest before or upon the termination of the preceding freehold estate.Example: O conveys "to A for life, then to B if she reaches age 21." If A dies before B reaches age 21, B's remainder is destroyed.Most states have abolished the destructibility rule. In those states, B's interest in the above example would be converted to an executory interest upon A's death because it will divest O's reversionary estate when B turns 21. a) Related Doctrine of MergerWhen one person acquires all of the present and future interests in land except a contingent remainder, under the common law, the contingent remainder is destroyed.Example: O conveys "to A for life, then to B's children." If, before B has any children, O purchases A's life estate, O will have a life estate pur autre vie and a reversion. These interests merge, and the contingent remainder in B's unborn children is destroyed.EXAM TIPWhen considering whether estates merge to destroy a contingent remainder, remember that if the life estate and the next vested interest were created by the same instrument, there is no merger. (This would defeat the grantor's obvious intent.) Merger may occur only as in the example above, when one person later acquires immediately successive estates. e. Rule in Shelley's Case (Rule Against Remainders in Grantee's Heirs)At common law, if the same instrument created a life estate in A and gave the remainder only to A's heirs, the remainder was not recognized, and A took the life estate and the remainder. Example: O conveys "to A for life, then to B for life, then to the heirs of A." The Rule transforms the remainder in A's heirs into a remainder in A. (No merger, however, because the remainder for life in B is vested.)The Rule in Shelley's Case has been abolished in most states.f. Doctrine of Worthier Title (Rule Against Remainders in Grantor's Heirs)Under the Doctrine of Worthier Title ("DOWT"), a remainder in the grantor's heirs is invalid and becomes a reversion in the grantor.Example: O grants Blackacre "to A for life, then to the heirs of O." Under DOWT, A has a life estate, and O has a reversion.DOWT is generally treated as a rule of construction (i.e., it does not apply if an intent to create a remainder in heirs has been clearly manifested). DOWT applies only to inter vivos transfers (not wills), and only if the word "heirs" is used. CHART 3. Executory InterestsExecutory interests are future interests in third parties that either divest a transferee's preceding freehold estate ("shifting interests"), or follow a gap in possession or cut short a grantor's estate ("springing interests").Examples: 1) In a grant from O "to A and his heirs when A marries B," A has a springing executory interest because it divests the grantor's estate.2) In a grant from O "to A for life, then to B and his heirs); but if B predeceases A, then to C and his heirs," C has a shifting executory interest because it divests a transferee's preceding estate.Executory interests are not considered vested and thus are subject to the Rule Against Perpetuities, but executory interests are not destructible.EXAM TIPRemember that if a third party's future interest does not follow the natural termination of the preceding estate, it must be an executory interest); a remainder cannot follow a fee simple estate. 4. Transferability of Remainders and Executory InterestsVested remainders are fully transferable, descendible, and devisable. At common law, contingent remainders and executory interests were not transferable inter vivos, but most courts today hold that they are freely transferable. Contingent remainders and executory interests are descendible and devisable, provided survival is not a condition to the interest's taking.EXAM TIPAny future interest that is transferable is subject to involuntary transfer); i.e., it is reachable by creditors.5. Class GiftsA "class" is a group of persons having a common characteristic (e.g., children, nephews). The share of each member is determined by the number of persons in the class. A class gift of a remainder may be vested subject to open (where at least one group member exists) or contingent (where all group members are unascertained). a. When the Class Closes - The Rule of ConvenienceUnder the rule of convenience, in the absence of express contrary intent, a class closes (i.e., no one born after that time may share in the gift) when some member of the class can call for distribution of her share of the class gift.Examples: 1) T's will devises property to W for life, then to A's children. At the time the will is executed, A has two children, B and C. A then has another child, D. T dies. A has child E, then W dies. After W's death, A has another child, F. The class closed at W's death because it was time to make the distribution. Thus, B, C, D, and E share the property, and F is excluded.2) T's will devises the residue of his estate "to those of A's children who attain age 21." If any of A's children is 21 at T's death, the class closes at that time. Otherwise it closes when one of A's children reaches age 21. But remember, if it had been a future gift (i.e., "to A for life, then to such of A's children who attain age 21"), the class would remain open until the life tenant's death even if some of the class members had reached the stated age at T's death.EXAM TIPRecall that persons in gestation at the time the class closes are included in the class.b. SurvivalSurvival of a class member to the time of closing is usually unnecessary to share in a future gift - unless survival was made an express condition (e.g., "to A for life and then to his surviving children"). However, certain terms are construed to create implied survivorship conditions (e.g., widow, issue, heirs, next of kin). CHART EXAM TIPGenerally, when the instrument creating a gift of a future interest in an open class becomes effective, existing class members have a vested remainder subject to open. But watch for a condition precedent, which will prevent the remainder from vesting. For example, "to A for life, remainder to those of B's children who survive A" creates a contingent remainder in B's children even if they are in existence - and even if B is dead - because the remainder is contingent on surviving A. C. TRUSTSA trust is a fiduciary relationship with respect to specific property (res) wherein the trustee holds legal title to the property subject to enforceable equitable rights in a beneficiary. The creator of a trust is the settlor,who must own the property at the time of trust creation and must have had the intent to create the trust. 1. Application of Rule Against PerpetuitiesThe Rule Against Perpetuities applies to the equitable future interests of the beneficiaries in a private trust just as it does to "legal" future interests.2. Creation of TrustsA trust can be created by will (testamentary trust), inter vivos transfer of the trust res, or inter vivos declaration that the settlor is holding property in trust. All trusts of real property must be in writing. Note that a settlor may bequeath (by will) property to a trust created during his lifetime - i.e., he may "pour it over" into the trust.3. Charitable TrustsA charitable trust must have a charitable purpose. The rules governing charitable trusts differ from those applicable to private trusts in three important ways: (i) a charitable trust must have indefinite beneficiaries); (ii) it may be perpetual (i.e., the Rule Against Perpetuities does not apply)); and (iii) the cy pres doctrine, which allows a court to select an alternative charity when the purpose of the settlor becomes impracticable or impossible, applies. Charitable trusts may be enforced by an action of the attorney general of the state.EXAM TIPRemember that the Rule Against Perpetuities does apply to a shift from a private to charitable use or a charitable to private use. D. THE RULE AGAINST PERPETUITIESNo interest in property is valid unless it must vest, if at all, not later than 21 years after some life in being ("measuring life") at the creation of the interest. If there is any possibility that the interest might vest more than 21 years after a life in being, the interest is void. The Rule applies to contingent remainders, executory interests, vested remainders subject to open (class gifts), options to purchase (not attached to a leasehold), rights of first refusal, and powers of appointment. 1. When Perpetuities Period Begins to RunThe time the interest is created and the perpetuities period begins to run depends on the instrument and the interest created: For interests granted by will, it runs from the date of the testator's death); for deeds, it is the date of delivery. The period runs on an irrevocable trust from the date it is created); it runs on a revocable trust from the date it becomes irrevocable.2. "Must Vest"An interest vests for purposes of the Rule when it becomes: (i) possessory, or (ii) an indefeasibly vested remainder or a vested remainder subject to total divestment.EXAM TIPIn analyzing Rule Against Perpetuities problems, keep in mind that the key is when the interest could possibly vest - not when it is likely to vest or even when it did. You must examine the grant as of the time of its creation and be sure that if the interest vests it will be within the period of the Rule (i.e., life in being plus 21 years). If there is any possibility that it could vest beyond the period, it is void.3. "Lives in Being"Unless other measuring lives are specified, one connected with the vesting of the interest is used. Any lives may be denominated measuring lives, provided they are human and of reasonable number.4. Interests Exempt from RuleExcept for vested remainders subject to open, the Rule Against Perpetuities does not apply to vested interests. Thus, other vested remainders, reversions, possibilities of reverter, and rights of entry are not subject to the Rule. Moreover, there is a charity-to-charity exception to the Rule (i.e., the Rule does not apply to any disposition over from one charity to another), and an exception for options to purchase held by a current tenant.EXAM TIPRemember that the Rule Against Perpetuities applies only to contingent remainders, executory interests, vested remainders subject to open, and in most states, options to purchase. Thus, the grantor's interests (reversions, possibilities of reverter, rights of entry) are safe from the Rule); you don't need to consider them.5. Consequence of Violating Rule - Offensive Interest StrickenViolation of the Rule destroys only the offending interest. The exception is the rare case of "infectious invalidity" where the testator would probably have preferred the entire gift to fail.6. The Rule in Operation - Common Pitfall Cases a. Executory Interest Following Defeasible FeeGenerally, an executory interest that follows a defeasible fee (e.g., "to A for so long as no liquor is consumed on the premises, then to B") violates the Rule Against Perpetuities, and the executory interest is stricken. (An executory interest following a defeasible fee is valid only if the condition is specific to the fee holder or expressly limited to the perpetuities period.)EXAM TIPWhen a void interest is stricken, the interests are classified as if the void interest were never there. For example, if O conveys "to A for as long as no liquor is consumed on the premises, then to B," B's interest would be stricken, A would have a fee simple determinable, and O would have a possibility of reverter. In contrast, if O conveys "to A, but if liquor is ever consumed on the premises, then to B," B's interest and the condition are stricken, and A has a fee simple absolute.b. Age Contingency Beyond Age Twenty-One in Open ClassA gift to an open class conditioned on members surviving beyond age 21 violates the Rule.Example: "To A for life, then to those of A's children who attain the age of 25." The remainder in A's children violates the Rule and is void.Some states have enacted perpetuities reform legislation that reduces such age contingencies to 21.c. Fertile OctogenarianA woman is conclusively presumed to be capable of bearing children, regardless of her age or medical condition.Example: "To A for life, then to A's children for life, then to A's grandchildren in fee." The remainder in A's grandchildren is invalid despite the fact that A is 80 years old.A few states have enacted perpetuities reform statutes that raise a presumption that women over a certain age (e.g., 55) cannot bear children. Also, medical testimony regarding a woman's childbearing capacity is admissible in these states.d. Unborn Widow or WidowerBecause a person's widow (or widower) is not determined until his death, it may turn out to be someone who was not in being at the time of the disposition.Example: O conveys "to A for life, then to A's widow for life, then to A's surviving issue in fee." In the absence of a statute to the contrary, the gift to A's issue is invalid because A's widow might be a spouse who was not in being when the interest was created.Compare: A remainder "to A's children" would be valid because, unlike issue, they would be determined at A's death.Where necessary to sustain a gift, a few state statutes raise a presumption that any reference to a person's spouse, widow, or widower is to a person in being at the time of the transfer.e. Administrative ContingencyA gift conditioned on an administrative contingency (e.g., admission of will to probate) violates the Rule.Example: A gift "to my issue surviving at the distribution of my estate" is invalid because the estate might be administered beyond the period of the Rule.A few state reform statutes eliminate this problem by raising a presumption that the transferor intended that the contingency should occur, if at all, within 21 years.f. Options and Rights of First Refusal 1) OptionsGenerally, an option to purchase that is structured so that it might be exercised later than the end of the perpetuities period is void. (A significant minority of courts will construe the option as lasting only for a reasonable time.) Exception: The Rule Against Perpetuities does not apply to options to purchase held by the current lessee.Example: When O conveys Blackacre to A, he includes a clause in the deed that states, "A, his heirs, and assigns promise that upon finding a ready, willing, and able buyer for Blackacre, Blackacre will be offered to O, his heirs, or assigns on the same terms." This right of first refusal can be exercised well beyond a life in being plus 21 years, and thus violates the Rule.EXAM TIPWatch for a fact pattern on the exam where a tenant has an option to purchase beyond the perpetuities period. Remember that the Rule does not apply to such an option held by a current tenant or his assignee, but it does apply to a former tenant and to any party to whom the current tenant might transfer the option separately from the lease (in jurisdictions permitting such a transfer).2) Rights of First RefusalIn contrast, rights of first refusal in many states are presumed to be personal to the holder and therefore not subject to the Rule Against Perpetuities. In some states, however, rights of first refusal are subject to the Rule in the same manner as options. 7. Application of the Rule to Class Gifts a. "Bad-as-to-One, Bad-as-to-All" RuleIf the interest of any class member may vest too remotely, the whole class gift fails. For the class gift to vest, the class must be closedand all conditions precedent must be satisfied for every member.b. "Gift to Subclass" ExceptionEach gift to a subclass may be treated as a separate gift under the Rule.Example: "Income to A for life, then to A's children for their lives. Upon the death of each of A's children, the corpus is to be distributed to that child's issue, per stirpes." The gifts to each of A's children's issue are considered separately. Thus, the gifts to issue of A's children living at the time of the disposition are good, but the gifts to the issue of afterborn children of A violate the Rule and are void.c. Per Capita Gift ExceptionA gift of a fixed amount to each member of a class is not treated as a class gift under the Rule.Example: "$1,000 to each of my great-grandchildren, whether born before or after my death." This creates gifts to individuals, each of whom is judged separately under the Rule. 8. Statutory ReformsIn most states, statutes modify the Rule Against Perpetuities. "Wait and see" statutes determine an interest's validity upon the termination of the preceding life estate - if the interest actually vests or fails within the perpetuities period, it is good); if it does not, it is void. Some states have statutes dealing with the common pitfall cases (see supra). Other statutes, including the Uniform Statutory Rule Against Perpetuities, provide alternative vesting periods (e.g., 90 years). Some allow court reformation of invalid interests to carry out the donor's general intent (e.g., cy pres). These reforms are irrelevant for bar exam purposes unless referred to in the question. CHARTE. THE RULE AGAINST RESTRAINTS ON ALIENATIONGenerally, any restriction on the transferability of a legal (as opposed to equitable) interest is void. 1. Types of Restraints on AlienationThere are three types of restraints on alienation: (i) disabling restraints, under which attempted transfers are ineffective); (ii) forfeiture restraints, under which an attempted transfer forfeits the interest); and (iii) promissory restraints, under which an attempted transfer breaches a covenant. A disabling restraint on any legal interest is void.2. Restraints on a Fee SimpleAll absolute restraints on fee simple estates are void); thus, the grantee may freely transfer the property. However, forfeiture or promissory restraints on fee simple estates for a limited time and reasonable purpose may be upheld (e.g., a restraint limited to the joint lifetimes of co-owners as a reasonable way to ensure that neither will have to reside with a stranger). a. Discriminatory RestraintsJudicial enforcement of restraints prohibiting the transfer or use of property to or by a person of specified racial, religious, or ethnic group is discriminatory state action forbidden by the Fourteenth Amendment. Discriminatory restrictions may also violate the Fair Housing Act. 3. Restraints on a Life EstateForfeiture and promissory restraints on life estates are valid, but disabling restraints are void.EXAM TIPRemember that the Rule Against Restraints on Alienation applies only to legal interests. Restraints on the alienation of equitable interests (e.g., spendthrift clauses in trust instruments) are valid.4. Restraints on Future InterestsRestraints on vested future interests generally are valid to the extent that restraints on present interests of the same type are valid (e.g., forfeiture and promissory restraints on vested remainders for life are valid, but disabling restraints on vested remainders for life are void); see 3., supra).5. Other Valid Restraints on AlienationThe following are valid restraints on alienation:a. Reasonable restrictions in commercial transactions);b. Reasonable options and rights of first refusal); andc. Restrictions on assignment and sublease of leaseholds (e.g., requiring landlord's consent). F. CONCURRENT ESTATESAn estate in land can be held concurrently by several persons, all of whom have the right to enjoyment and possession of the land. 1. Joint TenancyA joint tenancy's distinguishing feature is the right of survivorship. When one joint tenant dies, the property is freed from her concurrent interest (her survivors do not succeed to it). a. CreationThe common law requires four unities - time, title, interest, possession - to create a joint tenancy); i.e., the interests of joint tenants must be equal in every way. They must take identical interests, at the same time, by the same instrument, with the same right to possession. Thus, all interests in a joint tenancy must be equal shares. If there are three joint tenants, they each own an undivided one-third interest. In a tenancy in common, by contrast, equal shares are presumed, but are not required. For example, in a tenancy in common held by three parties, one tenant may own a two-thirds undivided interest while each of the other two tenants holds an undivided one-sixth share. In addition, modern law requires a clear expression of a right of survivorship); otherwise a conveyance to two or more persons is presumed to be a tenancy in common.EXAM TIPIf the bar examiners tell you in the question that the parties are joint tenants, take it as given that they are joint tenants with right of survivorship. In this situation, do not apply the presumption that any conveyance to two or more persons is a tenancy in common. The bar examiners are not testing your knowledge of that presumption unless the fact pattern actually gives you the quoted language of the grant creating the concurrent estate and asks you about the type of tenancy involved.b. SeveranceUnder certain circumstances, the right of survivorship is severed (i.e., terminated) and a tenancy in common results. 1) Inter Vivos ConveyanceA voluntary or involuntary conveyance by a joint tenant of her undivided interest destroys the joint tenancy. The transferee takes as a tenant in common. When there are more than two joint tenants, conveyance by one destroys the joint tenancy only to the extent of the conveyor's interest. a) Transactions that May Not Result in Severance (1) Judgment LiensUsually when a plaintiff obtains a money judgment against a defendant, that judgment becomes a lien on the defendant's real property in the county where the judgment is docketed. The lien runs with the land, burdening it until the judgment is paid or the lien expires (usually 10 years). If such a lien is acquired against a joint tenant, it does not sever the joint tenancy until it is actually sold at a foreclosure sale.(2) MortgagesIn most states, a mortgage is a lien on title and does not sever a joint tenancy. Severance occurs only if the mortgage is foreclosed and the property is sold. The execution of a mortgage in title theory states, however, does sever a joint tenancy.(3) LeasesStates are split as to whether one joint tenant's lease of her interest causes a severance. 2) Contract to ConveySeverance results if one joint tenant contracts to convey her interest, but the courts are split on whether an executory contract by all joint tenants works a severance. CHART 3) Testamentary Disposition Has No EffectA will is ineffective to work a severance because at death the testator's interest vanishes.4) Effect of One Joint Tenant's Murdering AnotherConceptually, a joint tenant who murders the other joint tenant should not lose her right of survivorship. In some jurisdictions, statutes change this result); in others, a constructive trust is imposed for the decedent's estate. 2. Tenancy by the EntiretyA tenancy by the entirety is a marital estate akin to joint tenancy. In some common law jurisdictions, it arises presumptively in any conveyance to two spouses. Only death, divorce, mutual agreement, or execution by a joint creditor of both the spouses can sever a tenancy by the entirety. An individual spouse cannot convey or encumber tenancy by the entirety property. A deed or mortgage executed by only one spouse is ineffective.3. Tenancy in CommonA tenancy in common is a concurrent estate with no right of survivorship. Tenants can hold different interests in the property, but each is entitled to possession of the whole. Interests are alienable, devisable, and inheritable. Today, multiple grantees are presumed to take as tenants in common, not as joint tenants.4. Rights and Duties of Co-Tenants a. PossessionEach co-tenant has the right to possess all portions of the property but has no right to exclusive possession of any part. A co-tenant out of possession cannot bring a possessory action unless she is "ousted" (e.g., another co-tenant claims right to exclusive possession).b. Rents and ProfitsIn most states, a co-tenant in possession has the right to retain profits from her own use of the property); i.e., she need not share profits with other co-tenants absent ouster or an agreement to the contrary. She must, however, share net rents from third parties and net profits gained from exploitations of land, such as mining.c. Effect of One Concurrent Owner's Encumbering the PropertyA joint tenant or tenant in common may encumber her interest (e.g., by mortgage or judgment lien), but may not encumber the interests of other co-tenants. If, e.g., one tenant in common mortgages her interest, the mortgagee can foreclose only on the mortgaging co-tenant's interest. If a joint tenancy is involved, a mortgage (in a lien theory state) or lien does not sever the joint tenancy, but a foreclosure sale will. Note, however, that in the case of a joint tenancy, a mortgagee or lienor runs the risk that the obligated co-tenant will die before foreclosure, extinguishing the mortgagee's or lienor's interest.d. Remedy of PartitionAny co-tenant has a right to judicial partition, either in kind (physical division of land among co-tenants) or by sale and division of the proceeds. Courts prefer partition in kind but will permit partition by sale when a fair and equitable physical division of the property cannot be made. Although generally this right may be exercised at any time, restraints on partition by co-tenants are valid, provided they are limited to a reasonable time.e. Expenses for Preservation of Property - Contribution 1) RepairsA co-tenant who pays more than her pro rata share of necessary repairs is entitled to contribution from the other co-tenants, provided she has notified the other co-tenants of the need for repairs.2) ImprovementsThere is no right of contribution for the cost of improvements unless there is a partition.3) Taxes and MortgagesContribution can be demanded for taxes or mortgage payments paid on the entire property. However, reimbursement to a co-tenant in sole possession is limited to the extent that expenditures exceed the rental value of her use. f. Duty of Fair DealingA confidential relationship exists among co-tenants); e.g., one co-tenant's acquisition of an outstanding title or lien that may affect the estate is deemed to be on behalf of other co-tenants. It is difficult for one co-tenant to adversely possess against other co-tenants.

Which future interest divests the interest of another transferee? AShifting executory interest BVested remainder subject to total divestment CSpringing executory interest

A A shifting executory interest is a future interest that divests the interest of another transferee. A conveyance from "O to A for life, then to B; but if B predeceases A, to C" creates a life estate in A, a vested remainder subject to total divestment in B (see below), and a shifting executory interest in C, because C's interest would divest the interest of B (another transferee). A vested remainder subject to total divestment does not divest the interest of another transferee. A remainder is a future interest that is created in a transferee and is capable of becoming a present interest upon the natural termination of the preceding estate. Generally, a remainder follows a life estate. A vested remainder subject to total divestment is a remainder created in an ascertained person in being whose interest is not subject to a condition precedent but is subject to being defeated by the happening of some condition subsequent. In the above example, B has a vested remainder subject to total divestment because B is an ascertained person in being whose interest is subject to being divested if she does not survive A. Although B's interest will become possessory upon the expiration of A's life estate, C's interest does not await the expiration of B's vested remainder, but instead may cut it short. Thus, C has a shifting executory interest. A springing executory interest does not divest the interest of another transferee. A springing executory interest is a future interest that follows a gap in possession or divests the estate of the transferor. A conveyance from "O to A for life, and one year after A's death to B" creates a life estate in A, a reversion in O (the transferor), and a springing executory interest in B, because it springs out of O's reversion.

Which future interest follows a gap in possession or divests the estate of the transferor? ASpringing executory interest BShifting executory interest CVested remainder subject to total divestment

A A springing executory interest is a future interest that follows a gap in possession or divests the estate of the transferor. A conveyance from "O to A for life, and one year after A's death to B" creates a life estate in A, a reversion in O (the transferor), and a springing executory interest in B, because it springs out of O's reversion. A vested remainder subject to total divestment does not follow a gap in possession or divest the estate of the transferor. A remainder is a future interest that is created in a transferee and is capable of becoming a present interest upon the natural termination of the preceding estate. Generally, a remainder follows a life estate. A vested remainder subject to total divestment is a remainder created in an ascertained person in being whose interest is not subject to a condition precedent but is subject to being defeated by the happening of some condition subsequent. A conveyance from "O to A for life, then to B; but if B predeceases A, to C" creates a life estate in A, a vested remainder subject to total divestment in B (because B is an ascertained person in being whose interest is subject to being divested if she does not survive A), and a shifting executory interest in C (see below). A shifting executory interest does not follow a gap in possession or divest the estate of the transferor. A shifting executory interest is a future interest that divests the interest of another transferee. In the above example, C has a shifting executory interest because it would divest the interest of B (another transferee).

Which tenancy continues for a fixed period of time and then automatically terminates without the landlord or the tenant giving notice? ATenancy for years BPeriodic tenancy CTenancy at sufferance DTenancy at will

A A tenancy for years continues for a fixed period of time and then automatically terminates without the landlord or the tenant giving notice. A periodic tenancy continues from period to period until terminated by proper notice by either the landlord or the tenant. The termination date of a periodic tenancy is uncertain until notice is given. A tenancy at will continues until terminated by the landlord or the tenant. Sufficient notice must be given to allow the tenant a reasonable time to quit the premises. Such a tenancy also terminates by operation of law if:1. Either party dies;2. The tenant commits waste;3. The tenant attempts to assign his tenancy;4. The landlord transfers her interest in the property; or5. The landlord executes a term lease to a third person. A tenancy at sufferance arises when a tenant wrongfully remains in possession after the expiration of a lawful tenancy. It lasts until either the landlord evicts the tenant or the tenant becomes a periodic tenant by virtue of the landlord's declaring him so or accepting additional rent.

Which of the following acts will not sever a joint tenancy? ATestamentary disposition by one joint tenant. BInter vivos conveyance by one joint tenant. CSuit for partition by one joint tenant. DIn a title theory state, the execution of a mortgage by one joint tenant.

A A testamentary disposition by one joint tenant will not sever a joint tenancy. A will devising a joint tenant's interest to another is inoperative as to joint tenancy property because when the testator dies (i.e., when the will becomes effective), his rights in the joint tenancy property are extinguished, and the will has no effect on them. A suit for partition by one joint tenant will sever a joint tenancy. The court will either divide the tract into parcels (partition in kind) or sell the property and divide the proceeds among the joint tenants in accordance with their ownership interests (partition by sale). An inter vivos conveyance by one joint tenant will sever a joint tenancy. The transferee takes the interest as a tenant in common and not as a joint tenant. In a title theory state, the execution of a mortgage by one joint tenant will sever a joint tenancy. In states following the lien theory, a mortgage is regarded as a lien on title, and one joint tenant's execution of a mortgage on her interest does not by itself cause a severance. But in states following the title theory, a mortgage is regarded as a transfer of title, and the transfer destroys the unity of title and severs the joint tenancy.

What interest arises when the remainderman is in existence and ascertained, but her right to possession and enjoyment is subject to being defeated by the happening of a condition subsequent? AVested remainder subject to total divestment BVested remainder subject to partial divestment CContingent remainder

A A vested remainder subject to total divestment arises when the remainderman is in existence and ascertained, but her right to possession and enjoyment is subject to being defeated by the happening of a condition subsequent. A condition subsequent is an express condition that, if it occurs, will divest the remainderman of her interest. A vested remainder subject to partial divestment (also called a vested remainder subject to open) arises when the remainderman is in existence and ascertained, but her interest is subject to diminution by reason of other persons becoming entitled to share in the remainder (i.e., a class gift). A contingent remainder is a remainder that is (i) subject to a condition precedent, or (ii) created in favor of unborn or unascertained persons. A condition precedent is an express condition that must occur before the remainder becomes possessory.

At common law, which of the following acts would subject a life tenant to liability for waste? AThe cutting of mature trees to clear the land for cultivation. BThe extraction of minerals from the land under a mineral lease. CThe cutting of mature trees for necessary repairs on the land. DThe extraction of minerals from a mine that was active prior to the grant.

A At common law, the cutting of mature trees to clear the land for cultivation would subject a life tenant to liability for waste. Voluntary waste occurs when a life tenant consumes or exploits natural resources on the property (e.g., timber, minerals, oil). Ameliorative waste consists of acts that economically benefit the property. Although clearing the land for cultivation may economically benefit the property, the life tenant would be liable for voluntary and/or ameliorative waste. At common law, the cutting of mature trees for necessary repairs on the land would not subject a life tenant to liability for waste. As explained above, a life tenant generally may not consume or exploit natural resources on the property. However, there are exceptions to this rule, one of which allows a life tenant to consume or exploit natural resources in reasonable amounts where necessary for repair and maintenance of the land. At common law, the extraction of minerals under a mineral lease would not subject a life tenant to liability for waste. Another exception to the general rule allows a life tenant to consume or exploit natural resources when she is expressly given the right, as in a lease. At common law, the extraction of minerals from a mine that was active prior to the grant would not subject a life tenant to liability for waste. Under the open mines doctrine, if a mine was active when the life estate began, the life tenant may continue to mine the property. However, the life tenant may not open any new mines.

If L leases property to T, and L subsequently assigns L's interest to L2, whom may T hold liable when X, a paramount title holder, ejects T? AL or L2 BL only CNeither L nor L2 DL2 only

A If L leases property to T, and L subsequently assigns L's interest to L2, T may hold L or L2 liable when X, a paramount title holder, ejects T. A landlord may assign the rents and reversion interest that he owns. The assignee is liable to the tenants for performance of all covenants made by the original landlord in the lease, provided that those covenants run with the land. The original landlord also remains liable on all of the covenants he made in the lease. X's evicting T from the entire leased premises breaches the covenant of quiet enjoyment, which runs with the land. Thus, L and L2 are personally liable to T. L only is incorrect because L2, the assignee, is liable for all lease covenants that run with the land, and the covenant of quiet enjoyment runs with the land. L2 only is incorrect because L, the original landlord, also remains liable on all covenants in the original lease after assignment. Neither L nor L2 is incorrect because the original landlord (L) remains liable on all covenants in the original lease after assignment, and the assignee (L2) is liable for all lease covenants that run with the land, including the covenant of quiet enjoyment.

In which of the following situations may L not terminate T's lease? AT refuses to pay higher rent after it is increased by L because T reported L for housing code violations. BT refuses to refrain from growing marijuana in his apartment. CT ceases paying rent after L breaches her covenant to clean the carpets every six months. DT assigns his interest to T2 in violation of a covenant against assignment.

A If T refuses to pay higher rent after it is increased by L because T reported L for housing code violations, L may not terminate T's lease. A landlord may not penalize a tenant for exercising the legal right to report housing or building code violations. This includes terminating the lease, raising the rent, or reducing tenant services. Although a tenant's failure to pay rent generally allows the landlord to terminate the lease, T would have a retaliatory eviction defense for L's retaliation in increasing the rent. If T ceases paying rent after L breaches her covenant to clean the carpets every six months, L may terminate T's lease. At common law, covenants in a lease were independent of each other; i.e., one party's performance of his promise did not depend on the other party's performance of her promise. Thus, if one party breached a covenant, the other party could recover damages but still had to perform his promises. However, an exception exists for nonpayment of rent; in nearly all states, a landlord may terminate the lease if the tenant breaches his covenant to pay rent. Thus, if L breaches her covenant to clean the carpets every six months, T can sue for damages but may not refuse to pay his rent. Failure to pay rent allows L to terminate the lease. If T refuses to refrain from growing marijuana in his apartment, L may terminate T's lease. A tenant has a duty to not use the leased premises for illegal purposes. If he breaches this duty, and the landlord is not a party to the illegal use, the landlord may terminate the lease or obtain damages and injunctive relief. If T assigns his interest to T2 in violation of a covenant against assignment, L may terminate T's lease. Although a transfer in violation of a covenant against assignment is not void, the landlord may terminate the lease under the lease terms or a statute or may sue for damages

Which of the following statements is correct regarding covenants against assignment or sublease? AIf a landlord consents to one transfer that violates a covenant against assignment or sublease, he waives his right to avoid future transfers. BA covenant against assignment or sublease is an unreasonable restraint on alienation. CIf a tenant transfers her interest in violation of a covenant against assignment or sublease, the transfer is void. DA covenant against assignment prevents the tenant from subleasing her interest.

A If a landlord consents to one transfer that violates a covenant against assignment or sublease, he waives his right to avoid future transfers. This is the Rule in Dumpor's Case. The landlord may reserve the right to avoid future transfers, but such reservation must take place at the time of granting consent. A covenant against assignment or sublease is NOT an unreasonable restraint on alienation. All jurisdictions permit and enforce such covenants. A covenant against assignment does NOT prevent the tenant from subleasing her interest. Covenants against assignment or sublease are strictly construed against the landlord. Thus, a covenant prohibiting assignment does not prohibit subleasing and vice versa. If a tenant transfers her interest in violation of a covenant against assignment or sublease, the transfer is NOT void. However, the landlord usually may terminate the lease under the lease terms or a statute or sue for damages.

If a residential tenant assuming possession of the leased premises discovers that the wiring is dangerously frayed and the plumbing is faulty, the landlord has breached the __________. Aimplied warranty of habitability Bcovenant against encumbrances Ccovenant of quiet enjoyment Dimplied covenant of marketability

A If a residential tenant assuming possession of the leased premises discovers that the wiring is dangerously frayed and the plumbing is faulty, the landlord has breached the implied warranty of habitability. Under the implied warranty of habitability for residential tenancies, the landlord covenants that the premises are suitable for human residence. The standard usually applied is the local housing code. Dangerous wiring and faulty plumbing likely violate the local housing code and will subject the landlord to liability for breach of the implied warranty of habitability. Under the covenant of quiet enjoyment, the landlord covenants that neither she nor someone with paramount title will interfere with the tenant's quiet enjoyment and possession of the premises. The covenant of quiet enjoyment may be breached by: total actual eviction, partial actual eviction, or constructive eviction. The implied covenant of marketability is not a landlord-tenant concept but is implied in every land sale contract. It obliges the seller to provide the buyer with marketable title (i.e., title reasonably free from doubt) at closing. The covenant against encumbrances is not a landlord-tenant concept but is a usual covenant contained in a general warranty deed. It is a covenant assuring that there are neither visible encumbrances (e.g., easements) nor invisible encumbrances (e.g., mortgages) against the title or interest conveyed.

In which of the following conveyances from O does charity A have a fee simple subject to an executory interest? A"To charity A, but if the premises are no longer used as a museum, to charity B" B"To charity A, provided that if the premises are no longer used as a museum, O may enter and terminate the estate" C"To charity A while the premises are used as a museum" D"To charity A"

A In a conveyance from O "to charity A, but if the premises are no longer used as a museum, to charity B," charity A has a fee simple subject to an executory interest. A fee simple subject to an executory interest is an estate that automatically divests in favor of a third party, rather than the grantor, on the happening of a stated event. The words "but if the premises are no longer used as a museum" create a condition on which charity A's interest will be terminated and a third party (charity B) will get possession of the land; hence, charity A's estate is subject to an executory interest in favor of charity B, which would be referred to simply as a fee simple subject to an executory interest. Charity B is not the grantor, and thus its interest is not a right of entry (the correlative future interest in a grantor who conveys a fee simple subject to a condition subsequent). Its interest is an executory interest because it divests a fee simple. Although executory interests are subject to the Rule Against Perpetuities, charity B's interest does not violate the Rule because the charity-to-charity exception provides that a gift over from one charity to another charity on a remote condition is valid. In a conveyance from O "to charity A while the premises are used as a museum," charity A does not have a fee simple subject to an executory interest. Rather, charity A has a fee simple determinable. A fee simple determinable is an estate that automatically terminates on the happening of a stated event and reverts to the grantor. It is created by the use of durational, adverbial language, such as "for so long as," "while," "during," or "until." In this conveyance, charity A has a fee simple because the estate may last forever if the premises are used as a museum. If, however, the premises cease to be used as a museum, the estate will automatically terminate and possession of the land will revert to O (rather than a third party). In a conveyance from O "to charity A, provided that if the premises are no longer used as a museum, O may enter and terminate the estate," charity A does not have a fee simple subject to an executory interest. Rather, charity A has a fee simple subject to a condition subsequent. A fee simple subject to a condition subsequent is an estate that terminates when the grantor exercises her power of termination (right of entry) after the happening of a stated event. Words such as "upon condition that," "provided that," "but if," and "if it happens that" are usually held to create conditions subsequent, and under most decisions the grantor must expressly reserve the right of entry. In this conveyance, if the premises cease to be used as a museum, charity A's estate will not end automatically but will continue until O asserts her right of entry. In a conveyance from O "to charity A," charity A does not have a fee simple subject to an executory interest. Rather, charity A has a fee simple absolute. A fee simple absolute is the largest estate permitted by law and is not subject to termination by the happening of a future event.

In the case of a will, the perpetuities period begins to run on the date: AThe testator dies BThe will is admitted to probate CThe will is executed

A In the case of a will, the perpetuities period begins to run on the date the testator dies. The validity of interests under the Rule Against Perpetuities is determined at the time the interests are created. The interests in a will are created when the will becomes effective, which is on the testator's death. Thus, the perpetuities period does not begin to run on the date the will is executed or admitted to probate

In the majority of states, if a hold-over tenant is occupying the premises when a new tenant's term begins, who must evict the hold-over tenant? AThe landlord BThe new tenant CNo one, because the hold-over tenant is a tenant at sufferance

A In the majority of states, if a hold-over tenant is occupying the premises when a new tenant's term begins, the landlord must evict the hold-over tenant. In most states, the landlord must deliver actual possession to the tenant at the beginning of the leasehold term. The landlord is in breach of this duty if she has not evicted a hold-over tenant by the beginning of the new tenant's term. In a minority of states, the landlord's obligation is merely to give the tenant the legal right to possession at the beginning of the leasehold term. In these states, if the premises are occupied by a hold-over tenant, it is up to the new tenant to bring eviction proceedings. Although the hold-over tenant is a tenant at sufferance, the landlord must evict the tenant and put the new tenant in actual possession of the premises. A tenant at sufferance is a tenant who wrongfully remains in possession after the expiration of a lawful tenancy. No notice is required to end the tenancy, which lasts only until the landlord takes steps to evict the tenant.

If a month-to-month tenant reports a landlord for housing code violations, may the landlord increase the tenant's rent the following month? ANo, if the landlord has a retaliatory motive BNo, because the rent increase would breach the implied warranty of habitability CYes, because the tenant may seek damages under the implied warranty of habitability DYes, if the landlord provides the tenant with adequate notice of the rent increase

A No, if a month-to-month tenant reports a landlord for housing code violations, the landlord may not increase the tenant's rent the following month if the landlord has a retaliatory motive; and a retaliatory motive is presumed for some period after the tenant reports the violations. A landlord may not penalize a tenant for exercising the legal right to report housing or building code violations. Retaliatory acts include terminating the lease, raising the rent, and reducing tenant services. Although a landlord generally may alter the terms of a periodic tenancy (e.g., raise rent) the following period if the landlord provides the tenant with adequate notice, the tenant is protected under the doctrine of retaliatory eviction if the landlord does so in retaliation against her reporting housing code violations. Under the implied warranty of habitability, the landlord covenants that the premises are suitable for human residence (e.g., up to the standards of the local housing code). Although the tenant may seek damages against a landlord who violates this warranty, a tenant who reports housing code violations may not be evicted in retaliation. Thus, the rent increase would NOT breach the implied warranty of habitability, but rather would be prohibited under the retaliatory eviction doctrine.

__________ waste results when a tenant intentionally or negligently damages the leased premises. AVoluntary BAmeliorative CPermissive

A Voluntary waste results when a tenant intentionally or negligently damages the leased premises. It also results when a tenant exploits minerals on the property, unless the property was previously so used or the lease provides that the tenant may do so. Permissive waste results when a tenant fails to make ordinary repairs to keep the leased premises in the same condition as at the commencement of the lease term, excluding ordinary wear and tear (unless the tenant covenanted to repair ordinary wear and tear). Ameliorative waste results when a tenant makes substantial alterations to the leased premises that increase the value of the property. Modern courts will permit a change in the character of premises that have significantly decreased in value over time if:1. The change increases the value of the premises;2. The change is performed by a long-term tenant (e.g., 25 years) or a life tenant; and3. The change reflects a change in the nature and character of the neighborhood. A. NATURE OF LEASEHOLDA leasehold is an estate in land, under which the tenant has a present possessory interest in the leased premises and the landlord has a future interest (reversion). 1. Tenancies for YearsA tenancy for years continues for a fixed period of time (e.g., L rents to T for two years). a. CreationTenancies for years are usually created by written leases. Under the Statute of Frauds, a writing is required if the lease is for more than one year.b. TerminationA tenancy for years ends automatically at its termination date. 1) Breach of CovenantsIn most leases, the landlord reserves aright of entry, which allows him to terminate the lease if the tenant breaches any of the lease's covenants. a) Failure to Pay RentIn many jurisdictions, a landlord may, by statute, terminate the lease upon the tenant's failure to pay the promised rent - even in the absence of a reserved right of entry. 2) SurrenderA tenancy for years may also terminate if the tenant surrenders the tenancy and the landlord accepts. The same formalities required for creation of the leasehold are required for surrender (e.g., if unexpired term exceeds one year, surrender must be in writing). 2. Periodic TenanciesA periodic tenancy continues for successive periods (e.g., month to month) until terminated by proper notice by either party. a. CreationA periodic tenancy can be created by:(i) Express agreement (e.g., L leases to T from month to month));(ii) Implication (e.g., L leases to T at a rent of $1,000 payable monthly)); or(iii) Operation of law (e.g., T remains in possession after the lease expires, and L treats it as a periodic tenancy); or the lease is invalid, but T goes into possession).b. TerminationA periodic tenancy is automatically renewed until proper notice of termination is given. Usually, the notice must be one full period in advance (e.g., one month's notice for a month-to-month tenancy) and timed to terminate the lease at the end of a period (e.g., the usual month-to-month tenancy can end only on the 30th or 31st, not the 15th). For a year-to-year lease, six months' notice is required. 3. Tenancies at WillA tenancy at will is terminable at the will of either the landlord or the tenant. a. CreationGenerally, a tenancy at will must be created by an express agreement that the lease can be terminated at any time. Absent such an agreement, periodic rent payments will cause a court to treat it as a periodic tenancy. If the lease gives only the landlord the right to terminate, a similar right will be implied in favor of the tenant. However, if only the tenant has a right to terminate, a similar right will not be implied in favor of the landlord.b. TerminationAlthough in some states a tenancy at will may be terminated with no notice by any party with the power to do so (see a., supra), in most states notice and a reasonable time to quit are required to terminate a tenancy at will. Alternatively, a tenancy at will may be terminated by operation of law (e.g., death, commission of waste, etc.). 4. Tenancies at Sufferance a. CreationA tenancy at sufferance arises when a tenant wrongfully remains in possession after the expiration of a lawful tenancy.b. TerminationA tenancy at sufferance lasts only until the landlord takes steps to evict the tenant. No notice of termination is required. CHART 5. The Hold-Over DoctrineIf a tenant continues in possession after his right to possession has ended, the landlord may: (i) evict him, or (ii) bind him to a new periodic tenancy. Generally, the terms and conditions of the expired tenancy govern the new one. Commercial tenants may be held to a new year-to-year tenancy if the original lease term was for one year or more, or a periodic term based on frequency of rent payments (e.g. month to month) if the original term was for less than one year. Residential tenants, however, are generally held to a new month-to-month tenancy, regardless of the original term. If the landlord notifies the tenant before the lease expires that occupancy after the termination will be at increased rent, the tenant, by holding over, is held to have acquiesced to the new terms (even if the tenant actually objected to the new terms).EXAM TIPThere are exceptions to the hold-over doctrine. Watch for situations where: (i) the tenant remains in possession for only a few hours after termination or leaves a few articles of personal property, (ii) the delay is not the tenant's fault (e.g., severe illness), or (iii) it is a seasonal lease. In these cases, the landlord cannot bind the tenant to a new tenancy. B. LEASESA lease is a contract that governs the landlord-tenant relationship. At common law, covenants in the lease were independent); i.e., if one party breached a covenant, the other party could recover damages but still had to perform his promises and could not terminate the landlord-tenant relationship. Today, the doctrines of actual and constructive eviction and the implied warranty of habitability are exceptions to this rule. Also, in nearly all states, the landlord may terminate the lease for the nonpayment of rent. Modern courts are also likely to construe other covenants as dependent and excuse one party's performance if the other party's breach relates to a material part of the lease (e.g., the Uniform Residential Landlord and Tenant Act ("URLTA"), which has been adopted in nearly half the states, codifies this rule for residential tenancies).C. TENANT DUTIES AND LANDLORD REMEDIES 1. Tenant's Duty to Repair (Doctrine of Waste)A tenant cannot damage (i.e., commit waste on) the leased premises. The rules governing waste in the leasehold context are much like those governing waste in the life estate context. a. Types of WasteThere are three types of waste: 1) Voluntary (affirmative) waste results when the tenant intentionally or negligently damages the premises or exploits minerals on the property.2) Permissive waste occurs when the tenant fails to take reasonable steps to protect the premises from damage from the elements. The tenant is liable for all ordinary repairs, excluding ordinary wear and tear. If the duty is shifted to the landlord (by lease or statute), the tenant has a duty to report deficiencies promptly.3) Ameliorative waste occurs when the tenant alters the leased property, thereby increasing its value. Generally, the tenant is liable for the cost of restoration. There is a modern exception to this rule, however, which permits a tenant to make this type of change if he is a long-term tenant and the change reflects changes in the neighborhood. b. Destruction of Premises Without FaultIf the leased premises are destroyed without the fault of either the landlord or the tenant, no waste is involved. In the absence of lease language or a statute to the contrary, neither party has a duty to restore the premises, but the tenant has a duty to continue paying rent. In most states, statutes or case law now give the tenant the option to terminate the lease in this situation, even in the presence of an explicit covenant to repair.c. Tenant's Liability for Covenants to RepairIf a residential tenant covenants to repair, the landlord usually remains obligated to repair (except for damages caused by the tenant) under the nonwaivable "implied warranty of habitability" (see D.3., infra). However, a nonresidential tenant's covenant to repair is enforceable, and a landlord may be awarded damages for breach based on the property's condition when the lease terminates compared with its condition when the lease commenced. A tenant who covenants to repair is not usually liable to rebuild after structural damage or casualty destruction, unless the covenant expressly includes these types of repairs. In the absence of a specific reference to ordinary wear and tear, a covenant to repair usually includes such repairs. However, repair covenants frequently exclude ordinary wear and tear. 2. Duty to Not Use Premises for Illegal PurposeIf the tenant uses the premises for an illegal purpose, the landlord may terminate the lease or obtain damages and injunctive relief. Occasional unlawful conduct by the tenant does not breach this duty.3. Duty to Pay RentAt common law, rent was due at the end of the leasehold term. However, leases usually contain a provision making rent payable at some other time (e.g., "monthly in advance"). Most states today have statutes providing that if the leasehold terminates before the time originally agreed upon, the tenant must pay a proportionate amount of the agreed rent. a. Rent DepositsThe landlord is not permitted to retain a security deposit beyond the damages actually suffered. If a rent deposit is denominated a "bonus," the landlord can retain it after the tenant is evicted. Most states restrict the amount of security deposits to one month's rent, require landlords to pay interest on security deposits, and allow statutory or punitive damages for a landlord's improper refusal to return a security deposit. Clauses in leases that attempt to avoid these state laws are void.b. Termination of Rent Liability - SurrenderIf a tenant effectively conveys (surrenders) his leasehold interest back to the landlord, his duty to pay rent ends. 4. Landlord Remedies a. Tenant on Premises But Fails to Pay Rent - Evict or Sue for RentAt common law, a breach of the lease, such as failure to pay rent, resulted only in a cause of action for money damages); a breach did not give rise to a right to terminate the lease. Most modern leases, however, give the nonbreaching party the right to terminate. Thus, if a tenant is on the premises and fails to pay rent, the landlord may bring suit for rent due or may evict the tenant under the state's unlawful detainer statute. The only issue in an unlawful detainer proceeding is whether the tenant has the right to possession); the tenant cannot raise counterclaims. Also, some states allow a landlord who does not receive rent to assert a lien on personal property found on the premises.b. Tenant Abandons - Do Nothing or RepossessIf the tenant unjustifiably abandons the property, the majority view is that the landlord has a duty to mitigate damages by seeking to relet the premises. If the landlord repossesses and/or relets, the tenant's liability depends on whether the landlord has accepted the surrender. If surrender is not found, the tenant is liable for the difference between the promised rent and the fair rental value of the property (in cases of reletting, between the promised rent and the rent received from the reletting). If surrender is found, the tenant is free from any rent liability accruing after abandonment. Note that the landlord's resumption of possession for herself constitutes acceptance of surrender. However, if the landlord has a duty to mitigate, she must repossess and attempt to relet); this will not constitute acceptance of surrender. D. LANDLORD DUTIES AND TENANT REMEDIESAt common law, a landlord had no duty to repair or maintain the premises. This rule has been modified for residential tenancies in most states by statute (e.g., the URLTA) and the implied warranty of habitability. 1. Duty to Deliver Possession of PremisesStatutes in most states require the landlord to put the tenant in actual possession of the premises at the beginning of the leasehold term); i.e., the landlord is in breach if he has not evicted a hold-over tenant by the beginning of the lease term.2. Quiet EnjoymentEvery lease has an implied covenant that neither the landlord nor a paramount title holder (e.g., a prior mortgagee who forecloses) will interfere with the tenant's quiet enjoyment and possession of the premises. This covenant may be breached in the following ways: a. Actual EvictionActual eviction occurs when the landlord, a paramount title holder, or a hold-over tenant excludes the tenant from the entire leased premises. Actual eviction terminates the tenant's obligation to pay rent.b. Partial EvictionPartial actual eviction occurs when the tenant is physically excluded from only part of the leased premises. Partial eviction by the landlord relieves the tenant of the obligation to pay rent for the entire premises, even though the tenant continues in possession of the remainder. Partial eviction by a third person with paramount title results in an apportionment of rent); i.e., the tenant is liable for the reasonable rental value of the portion she continues to possess.c. Constructive EvictionConstructive eviction occurs when a landlord's breach of duty renders the premises unsuitable for occupancy. To establish a claim for constructive eviction, the tenant must prove:(i) The landlord breached a duty to the tenant);(ii) The breach substantially and materially deprived the tenant of her use and enjoyment of the premises (e.g., flooding, absence of heat in winter, loss of elevator service in a warehouse));(iii) The tenant gave the landlord notice and a reasonable time to repair); and(iv) After such reasonable time, the tenant vacated the premises.A tenant who has been constructively evicted may terminate the lease and may also seek damages.EXAM TIP:Remember that a tenant who remains on the premises cannot claim constructive eviction. If a tenant is occupying the premises despite poor conditions, he cannot sue the landlord for constructive eviction or raise it as a defense to a landlord's suit for rent. 3. Implied Warranty of HabitabilityMost jurisdictions imply a covenant of habitability into residential leases. This warranty is nonwaivable. The landlord's duty is tied to standards of local housing codes. In the event of a breach, the tenant may: (i) terminate the lease); (ii) make repairs and offset the cost against future rent); (iii) abate the rent to an amount equal to the fair rental value in view of the defects); or (iv) remain in possession, pay full rent, and sue for damages.EXAM TIPKeep in mind that the implied warranty of habitability does not apply to commercial tenants - only to residential tenants.4. Retaliatory EvictionIn many states, a landlord may not terminate a lease or otherwise penalize a tenant in retaliation for the tenant's exercise of her legal rights, including reporting housing or building code violations. Many statutes presume a retaliatory motive if the landlord acts within, e.g., 90 to 180 days after the tenant exercises her rights. To overcome the presumption, the landlord must show a valid, nonretaliatory reason for his actions.5. DiscriminationTenants and potential tenants are protected by the Civil Rights Act of 1866, which bars racial or ethnic discrimination, and the Fair Housing Act, which bars discrimination based on ethnicity, religion, national origin, gender, and disability, as well as discrimination against families with children (except in senior housing). The Act does not apply to religious organizations, private clubs, and owners who have no more than three single-family dwellings or who have an owner-occupied apartment with no more than four units. a. Reasonable AccommodationsWhen the Fair Housing Act applies, landlords must permit disabled tenants to make reasonable modifications to existing premises to accommodate their disabilities at the tenants' own expense. Landlords must also make reasonable accommodations in rules, policies, and services when necessary to afford a disabled person an equal opportunity to use a dwelling. E. ASSIGNMENTS AND SUBLEASESAbsent an express restriction in the lease, a tenant may freely transfer her leasehold interest, in whole or in part. A complete transfer of the entire remaining term is an assignment. If the tenant retains any part of the remaining term (other than a right to reenter upon breach), the transfer is a sublease.EXAM TIPFor bar exam purposes, a transfer will be considered a sublease, rather than an assignment, only when the original tenant reserves time for herself (e.g., the last month of the lease). 1. Consequences of AssignmentAn assignee stands in the shoes of the original tenant in a direct relationship with the landlord); i.e., the assignee and the landlord are in "privity of estate," and each is liable to the other on all covenants in the lease that "run with the land." a. Covenants that Run with the LandA covenant runs with the land if the original parties to the lease so intend and if the covenant "touches and concerns" the land (i.e., benefits the landlord and burdens the tenant (or vice versa) with respect to their interests in the property).b. Rent CovenantsBecause a covenant to pay rent runs with the land, the assignee owes rent directly to the landlord. After assignment, the original tenant is no longer in privity of estate with the landlord but remains liable on the original contractual obligation to pay rent (privity of contract). If the assignee reassigns the leasehold interest, his privity of estate with the landlord ends, and he has no liability for the subsequent assignee's failure to pay rent. 2. Consequences of Sublease - Sublessee Not in Privity with LandlordA sublessee is the tenant of the original lessee and usually pays rent to the original lessee, who then pays the landlord. A sublessee is not personally liable to the landlord for rent or for the performance of any of the covenants in the main lease unless the sublessee expressly assumes the covenants. a. Landlord's RemediesThe landlord may terminate the main lease for nonpayment of rent or breach of other covenants if the lease so states or the power is given by statute. The sublease automatically terminates with the main lease.b. Rights of SublesseeA sublessee cannot enforce any covenants made by the landlord in the main lease, except a residential sublessee may be able to enforce the implied warranty of habitability against the landlord. CHART 3. Covenants Against Assignment or SubleaseLease covenants restricting assignment and sublease are strictly construed against the landlord. (Thus, a covenant prohibiting assignment does not prohibit subleasing and vice versa.) a. WaiverA valid covenant against assignment is considered waived if the landlord was aware of the assignment and did not object (e.g.,by knowingly accepting rent from the assignee). Once the landlord consents to one transfer, the Rule in Dumpor's Case provides that he waives the covenant as to future transfers unless he expressly reserves it.b. Transfer in Violation of LeaseIf a tenant assigns or sublets in violation of a lease provision, the transfer is not void. The landlord, however, usually may terminate the lease or sue for damages. 4. Assignments by LandlordsA landlord may assign the rents and reversion interest he owns. This is usually done by deed when the landlord conveys a building to a new owner. The tenants' consent is not required. a. Rights of Assignee Against Tenants - AttornmentOnce tenants are given reasonable notice of the assignment, they must recognize and pay rent to the new owner as their landlord. The benefit of all tenant covenants that touch and concern the land runs with the landlord's estate to the new owner.b. Liabilities of Assignee to TenantsThe burden of the landlord's covenants that touch and concern the land runs with the landlord's estate to the assignee); thus, the assignee is liable for the performance of those covenants. The original landlord also remains liable on all of the covenants he made in the lease. F. CONDEMNATION OF LEASEHOLDSIf the entire leasehold is taken by eminent domain, the tenant's liability for rent is extinguished because both the leasehold and reversion have merged in the condemnor and there is no longer a leasehold estate. The lessee is entitled to compensation. However, if the taking is temporary or partial, the tenant is not discharged from the rent obligation, but is entitled to compensation (i.e., a share of the condemnation award) for the taking.G. TORT LIABILITY OF LANDLORD AND TENANT 1. Landlord's LiabilityAt common law, a landlord had no duty to make the premises safe. Today, there are six exceptions. a. Concealed Dangerous Condition (Latent Defect)If, at the time the lease is entered into, the landlord knows (or should know) of a dangerous condition that the tenant could not discover by reasonable inspection, the landlord must disclose (not repair) it. Otherwise, the landlord will be liable for any injuries resulting from the condition. If the tenant accepts the premises after disclosure, she assumes the risk for herself and others); the landlord is no longer liable.b. Common AreasThe landlord has a duty of reasonable care in maintaining common areas (e.g., halls, elevators).c. Public UseA landlord is liable for injuries to members of the public if, at the time of the lease, he:1) Knows (or should know) of a dangerous condition);2) Has reason to believe the tenant may admit the public before repairing the condition); and3) Fails to repair the condition.d. Furnished Short-Term ResidenceA landlord who rents a fully furnished premises for a short period (e.g., summer cottage) is under a stricter duty. He is liable for injuries resulting from any defect whether or not he knew of the defect.e. Negligent Repairs by LandlordEven if a landlord has no duty to make repairs, a landlord who actually attempts to repair is liable if an injury results because the repairs are done negligently or give a deceptive appearance of safety.f. Landlord Contracts to RepairIf the landlord covenants to repair, he is liable for injuries resulting from his failure to repair or negligent repair. 2. Modern Trend - General Duty of Reasonable CareMany courts are now holding that a landlord owes a general duty of reasonable care toward residential tenants, and will be held liable for injuries resulting from ordinary negligence if he had notice of a defect and an opportunity to repair it. a. Defects Arising After Tenant Takes PossessionA landlord generally is held to have notice of defects existing before the tenant took possession but is not liable for defects arising after the tenant takes possession unless the landlord knew or should have known of them.b. Legal Duty to RepairIf the landlord has a statutory duty to repair (e.g., housing codes), he is liable for injuries resulting from his failure to repair or negligent repair.c. SecuritySome courts hold landlords liable for injuries to tenants inflicted by third-party criminals where the landlord failed to comply with housing code provisions dealing with security, maintain ordinary security measures, or provide advertised extraordinary security measures (e.g., surveillance cameras). 3. Tenant's LiabilityThe duty of care owed by a tenant, as an occupier of land, to third persons is discussed in the Torts outline.

A __________ restraint provides that any attempted transfer of the property is ineffective. AForfeiture BDisabling CPromissory

B A disabling restraint provides that any attempted transfer of the property is ineffective. A conveyance from "O to A, and neither A nor her heirs shall have the right to transfer the land or any interest therein" contains a disabling restraint. Disabling restraints on any legal interest are void. A forfeiture restraint provides that if the grantee attempts to transfer the property, it is surrendered to another person. A conveyance from "O to A, but if A attempts to transfer the land or any interest therein during her lifetime, to B" contains a forfeiture restraint. Partial (i.e., limited in time and purpose) forfeiture restraints on legal interests generally are valid. A promissory restraint provides that the grantee covenants not to transfer the property. A conveyance from "O to A, and A hereby covenants not to transfer the land or any interest therein without O's consent" contains a promissory restraint. Partial promissory restraints on legal interests generally are valid.

A fee simple determinable is an estate that: ADivests in favor of a third person on the happening of a stated event BAutomatically terminates on the happening of a stated event and reverts to the grantor CContinues after the happening of a stated event until the grantor exercises her power of termination

B A fee simple determinable is an estate that automatically terminates on the happening of a stated event and reverts to the grantor. A conveyance from "O to A for so long as/while/during/until [event]" creates a fee simple determinable in A and a possibility of reverter in O. A fee simple determinable is not an estate that continues after the happening of a stated event until the grantor exercises her power of termination. An estate that continues after the happening of a stated event until the grantor exercises her power of termination (right of entry) is a fee simple subject to a condition subsequent. A conveyance from "O to A, upon condition that/provided that/but if/if it happens that [event], O may enter and terminate the estate" creates a fee simple subject to a condition subsequent in A and a right of entry in O. A fee simple determinable is not an estate that divests in favor of a third person on the happening of a stated event. An estate that terminates on the happening of a stated event and divests in favor of a third person, rather than in favor of the grantor, is a fee simple subject to an executory interest. A conveyance from "O to A for so long as/while/during/until [event], then to B" or "provided that/but if/if it happens that [event], then to B" creates a fee simple subject to an executory interest.

A __________ restraint provides that if the grantee attempts to transfer the property, it is surrendered to another person. ADisabling BForfeiture CPromissory

B A forfeiture restraint provides that if the grantee attempts to transfer the property, it is surrendered to another person. A conveyance from "O to A, but if A attempts to transfer the land or any interest therein during her lifetime, to B" contains a forfeiture restraint. Reasonable (i.e., limited in time and purpose) forfeiture restraints on legal interests generally are valid. A disabling restraint provides that any attempted transfer of the property is ineffective. A conveyance from "O to A, and neither A nor her heirs shall have the right to transfer the land or any interest therein" contains a disabling restraint. Disabling restraints on any legal interest are void. A promissory restraint provides that the grantee covenants not to transfer the property. A conveyance from "O to A, and A hereby covenants not to transfer the land or any interest therein without O's consent" contains a promissory restraint. Reasonable promissory restraints on legal interests generally are valid.

A grantor who conveys a fee simple subject to a condition subsequent retains __________. AA possibility of reverter BA right of entry CNo interest DA reversion

B A grantor who conveys a fee simple subject to a condition subsequent retains a right of entry. A conveyance from "O to A upon condition that/provided that/but if/if it happens that [event], then O or her heirs may enter and terminate the estate" creates a fee simple subject to a condition subsequent in A and a right of entry in O. A possibility of reverter is the future interest retained by a grantor who conveys a fee simple determinable. A conveyance from "O to A for so long as/while/during/until [event]" creates a fee simple determinable in A and a possibility of reverter in O. A reversion is the future interest retained by a grantor who conveys a lesser estate than the grantor owns. Where O has a fee simple, a conveyance from "O to A for life" creates a life estate in A and a reversion in O.

A grantor who conveys a lesser estate than the grantor owns retains __________. AA possibility of reverter BA reversion CNo interest DA right of entry

B A grantor who conveys a lesser estate than the grantor owns retains a reversion. For example, where O has a fee simple, a conveyance from "O to A for life" creates a life estate in A and a reversion in O. A possibility of reverter is the future interest retained by a grantor who conveys a fee simple determinable. A conveyance from "O to A for so long as/while/during/until [event]" creates a fee simple determinable in A and a possibility of reverter in O. A right of entry is the future interest retained by a grantor who conveys a fee simple subject to a condition subsequent. A conveyance from "O to A upon condition that/provided that/but if/if it happens that [event], then O or her heirs may enter and terminate the estate" creates a fee simple subject to a condition subsequent in A and a right of entry in O.

A tenancy for years is a tenancy that continues __________. Auntil terminated by the landlord or the tenant Bfor a fixed period of time until it automatically expires without the landlord or the tenant giving notice Cfrom year to year until terminated by proper notice by either the landlord or the tenant Duntil the landlord evicts the tenant

B A tenancy for years is a tenancy that continues for a fixed period of time until it automatically expires without the landlord or the tenant giving notice. A tenancy for years may be for more or less than a year, but it must have a fixed period or termination date. A tenancy that continues from year to year until terminated by proper notice by either the landlord or the tenant is a periodic tenancy. The termination date of a periodic tenancy is always uncertain until notice is given. A tenancy that continues until terminated by the landlord or the tenant is a tenancy at will. Sufficient notice must be given to allow the tenant a reasonable time to quit the premises. Such a tenancy also terminates by operation of law if: Either party dies;The tenant commits waste;The tenant attempts to assign his tenancy;The landlord transfers her interest in the property; orThe landlord executes a term lease to a third person. A tenancy that continues until the landlord evicts the tenant is a tenancy at sufferance. A tenancy at sufferance arises when a tenant wrongfully remains in possession of the leased premises after the expiration of a lawful tenancy. No notice is required to end the tenancy, which may not even be an estate in land

Absent a covenant to repair, liability for negligent repairs voluntarily undertaken by the landlord: AIs attributed to neither the landlord nor the tenant BIs attributed to the landlord CIs apportioned between the landlord and the tenant DIs attributed to the tenant

B Absent a covenant to repair, liability for negligent repairs voluntarily undertaken by the landlord is attributed to the landlord. Even if a landlord has no duty to make repairs, a landlord who actually attempts to repair is liable for injuries resulting from repairs that were made negligently or that give a deceptive appearance of safety. Liability for such negligent repairs is NOT attributed to the tenant or apportioned between the landlord and the tenant.

Which interest violates the Rule Against Perpetuities and is stricken if it follows a defeasible fee and has no limit on the time within which it must vest? AA contingent remainder BAn executory interest CA possibility of reverter

B An executory interest that follows a defeasible fee and has no limit on the time within which it must vest violates the Rule Against Perpetuities and is stricken. The effect on the remaining fee estate depends on whether the fee is determinable or subject to a condition subsequent. If the defeasible fee is phrased in durational terms (e.g., "for so long as," "until"), the estate still will terminate upon the happening of the stated event, and the grantor will have a possibility of reverter. In contrast, if the fee is subject to a condition subsequent, the condition is stricken as well, and the estate becomes a fee simple absolute. Although a contingent remainder is subject to the Rule Against Perpetuities, a remainder cannot follow a fee simple interest of any kind. Any interest that follows a fee and is held by a third person is an executory interest. A possibility of reverter is the future interest retained by a grantor who conveys a fee simple determinable (a defeasible fee). Future interests in the grantor are not subject to the Rule Against Perpetuities.

At common law, which of the following unities are required to create a joint tenancy? ATime, title, interest, and marriage BTime, title, interest, and possession CTitle, interest, possession, and right of survivorship

B At common law, the unities of time, title, interest, and possession are required to create a joint tenancy. The unity of time requires that the interest of each joint tenant vest at the same time. The unity of title requires that all joint tenants acquire title by the same instrument. The unity of interest requires that the interest of each joint tenant be of the same type and duration. The unity of possession requires that each joint tenant have the right to possession of the whole. If these four unities are not present, a joint tenancy cannot be created at common law. Instead, a tenancy in common results. A tenancy in common is a concurrent estate with no right of survivorship. Title, interest, possession, and right of survivorship are not the unities required to create a joint tenancy. Rather, the right of survivorship is a benefit of a joint tenancy. In a joint tenancy, each co-tenant owns an undivided share of the property, and the surviving co-tenant has the right to the whole estate (i.e., has the right of survivorship). Time, title, interest, and marriage are not the unities required to create a joint tenancy. A tenancy by the entirety is a marital estate akin to a joint tenancy in that four unities (plus a fifth—marriage) are required for its creation, and the surviving spouse has the right of survivorship. Marriage, however, is not required to create a joint tenancy.

If A, B, and C own property as joint tenants, and A conveys his interest to D, what interests do the parties own on B's death? AB's estate, C, and D each own a 1/3 interest BC owns a 2/3 interest, and D owns a 1/3 interest CC and D each own a 1/2 interest

B If A, B, and C own property as joint tenants, and A conveys his interest to D, on B's death C owns a 2/3 interest, and D owns a 1/3 interest. The distinguishing feature of a joint tenancy is the right of survivorship. When property is held by three or more joint tenants, one joint tenant's conveyance destroys the joint tenancy only as to that interest. The remaining joint tenants continue to hold in joint tenancy as between themselves, and the grantee holds his interest as a tenant in common with them. When A sold his interest to D, that 1/3 interest was severed and thus converted into a tenancy in common, which D continues to hold. However, the remaining 2/3 interest continues to be held in joint tenancy with right of survivorship between B and C. When one joint tenant dies, the property is freed from her interest, and the survivors retain an undivided right in the property. Since B's interest was extinguished on her death, B's estate does not take B's interest; the surviving joint tenant (C) holds free of it. Thus, C owns a 2/3 interest in the property. C and D do NOT each own a 1/2 interest. This would be the case if D took A's share as a joint tenant. However, as is explained above, D took as a tenant in common. Thus, D's share does not increase on B's death, because he does not have the benefit of the right of survivorship. B's estate, C, and D do NOT each own a 1/3 interest. As is explained above, B's estate does not take B's interest on B's death

If O conveys property "to A for life, then to B; but if B predeceases A, to C," what interest does B have? AContingent remainder BVested remainder subject to total divestment CVested remainder subject to open DIndefeasibly vested remainder

B If O conveys property "to A for life, then to B; but if B predeceases A, to C," B has a vested remainder subject to total divestment. A vested remainder subject to total divestment arises when the remainderman is in existence and ascertained and his interest is not subject to any condition precedent, but his right to possession and enjoyment is subject to being defeated by the happening of a condition subsequent. B, an ascertained person in being whose interest is not subject to a condition precedent, has a vested remainder, but his interest is subject to being divested if he does not survive A. B does not have an indefeasibly vested remainder. An indefeasibly vested remainder is a remainder that (i) can be created in and held only by an ascertained person in being, (ii) must be certain to become possessory on the termination of the prior estate, (iii) must not be subject to being defeated or divested, and (iv) must not be subject to being diminished in size. B is an ascertained person in being, but his vested remainder is subject to being divested if he does not survive A. B does not have a vested remainder subject to open. A vested remainder subject to open is a remainder created in a class of persons that is certain to take on the termination of the preceding estate, but is subject to diminution by reason of other persons becoming entitled to share in the remainder. B's vested remainder is not subject to partial divestment by other remaindermen. B does not have a contingent remainder. A contingent remainder is a remainder that is (i) subject to a condition precedent, or (ii) created in favor of unborn or unascertained persons. B is an ascertained person in being whose interest is not subject to a condition precedent.

If O conveys property "to A for life, then to B; but if B predeceases A, to C," what interest does C have? AVested remainder subject to total divestment BShifting executory interest CSpringing executory interest DAlternative contingent remainder

B If O conveys property "to A for life, then to B; but if B predeceases A, to C," C has a shifting executory interest. A shifting executory interest is a future interest that divests the interest of another transferee. A has a life estate. B has a vested remainder subject to total divestment because B is an ascertained person in being whose interest is not subject to a condition precedent but is subject to being divested if she does not survive A. Because C's interest may divest B's interest, it is a shifting executory interest. C does not have a vested remainder subject to total divestment. A remainder is a future interest created in a transferee that is capable of becoming a present interest upon the natural termination of the preceding estate. Generally, a remainder follows a life estate. If the interest is not capable of taking on the natural termination of the preceding estate, it is an executory interest. As explained above, B has a vested remainder subject to total divestment. C's interest does not await the expiration of B's vested remainder, but instead may cut it short. Thus, it is an executory interest. C does not have an alternative contingent remainder. A contingent remainder is a remainder that is (i) subject to a condition precedent, or (ii) created in favor of unborn or unascertained persons. As explained above, a remainder cannot divest a preceding estate prior to its normal expiration. Future interests that cut short a preceding estate are called executory interests. C does not have a springing executory interest. A springing executory interest is a future interest that follows a gap in possession or divests the estate of the transferor. If C's interest does take in present possession, it will divest the interest of another transferee (B) rather than the transferor (O), and title will shift to C.

If T devises property "to A for life, then to B's children," and B has at least one child living at all relevant times, when does the class of "B's children" close? AAt T's death BAt A's death CAt B's death DWhen the will is executed

B If T devises property "to A for life, then to B's children," and B has at least one child living at all relevant times, the class of "B's children" closes at A's death. Under the rule of convenience, a class closes when some member of the class can call for a distribution of her share of the gift. When possession and enjoyment of a gift are postponed, as where the gift follows a life estate, the class remains open until the time fixed for distribution (e.g., death of the life tenant). At T's death, the will becomes effective and A's life estate begins. At that time, B's child or children will have a vested remainder subject to open because the interest is certain to take on the termination of A's life estate but also could be partially divested by B's unborn children. At A's death, B's child or children will be entitled to distribution of the property, so the class will close at that time. Any children born to B after A's death will be excluded by the rule of convenience. If T devises property "to A for life, then to B's children," and B has at least one child living at all relevant times, the class of "B's children" does not close at T's death. As stated above, pursuant to the rule of convenience, when the beneficiaries of a postponed gift are a class, the class closes at the time fixed for distribution, not at the time the interest becomes effective. Although T's will becomes effective at T's death, the class does not close until B's child or children are entitled to distribution of the property—i.e., when A's life estate terminates (at A's death). If T devises property "to A for life, then to B's children," and B has at least one child living at all relevant times, the class of "B's children" does not close at B's death. As stated above, any children born to B after A's death will be excluded from the class by the rule of convenience. Note, however, that the rule of convenience will not be followed if contrary to the grantor's or testator's expressed intent. Thus, if instead T's will had said "to A for life, then to B's children, including children born to B after A dies," the class would remain open until B's death. If T devises property "to A for life, then to B's children," and B has at least one child living at all relevant times, the class of "B's children" does not close when the will is executed. The will does not become effective until T's death, and the time fixed for distribution is not until A's death. Thus, as explained above, the class remains open until A dies.

If T devises property "to B's children," and B has at least one child living at all relevant times, when does the class of "B's children" close? AAt B's death BAt T's death COn the date the will is executed

B If T devises property "to B's children," and B has at least one child living at all relevant times, the class of "B's children" closes at T's death. Under the rule of convenience, a class closes when some member of the class can call for a distribution of her share of the gift. When a will makes an outright gift to a class, if any class members are alive at the testator's death, the class closes as of the date of the testator's death. Here, because B has at least one child living at T's death, the class closes at T's death, and any children born to B after T's death will be excluded by the rule of convenience. If T devises property "to B's children," and B has at least one child living at all relevant times, the class of "B's children" does not close on the date the will is executed. As stated above, pursuant to the rule of convenience, when the beneficiaries of an outright gift are a class, the class closes when the will becomes effective. A will does not become effective until the testator's death. Thus, the class does not close on the date the will is executed. If T devises property "to B's children," and B has at least one child living at all relevant times, the class of "B's children" does not close at B's death. As stated above, any children born to B after T's death will be excluded from the class by the rule of convenience. Note, however, that if there are no members of the class living at the testator's death, all afterborn persons who come within the class designation are included. Thus, if B had no children living at T's death or within the period of gestation thereafter, then all of B's children, whenever born, would be included, and the class would close when B could have no more children—i.e., at B's death.

In which of the following conveyances from O does A have a fee simple determinable? A"To A and her heirs" B"To A for so long as the premises are used as an animal shelter" C"To A, but if the premises cease to be used as an animal shelter, O may enter and terminate the estate" D"To A for life until the premises cease to be used as an animal shelter"

B In a conveyance from O "to A for so long as the premises are used as an animal shelter," A has a fee simple determinable. A fee simple determinable is an estate that automatically terminates on the happening of a stated event and reverts to the grantor. It is created by the use of durational, adverbial language, such as "for so long as," "while," "during," or "until." A has a fee simple because the estate may last indefinitely if the premises are used as an animal shelter. If, however, the premises cease to be used as an animal shelter, the estate will automatically terminate and possession of the land will revert to O. In a conveyance from O "to A, but if the premises cease to be used as an animal shelter, O may enter and terminate the estate," A does not have a fee simple determinable. Rather, A has a fee simple subject to a condition subsequent. A fee simple subject to a condition subsequent is an estate that terminates when the grantor exercises her power of termination (right of entry) after the happening of a stated event. Words such as "upon condition that," "provided that," "but if," and "if it happens that" are usually held to create conditions subsequent, and under most decisions the grantor must expressly reserve the right of entry. In this conveyance, if the premises cease to be used as an animal shelter, A's estate will not end automatically but will continue until O asserts her right of entry. In a conveyance from O "to A for life until the premises cease to be used as an animal shelter," A does not have a fee simple determinable. Rather, A has a life estate determinable. A life estate is an estate that cannot last longer than the life or lives of one or more persons. It may be indefeasible (so that it will end only when the life tenant dies), or it may be made defeasible in the same way that fee estates can be defeasible (e.g., determinable, subject to a condition subsequent, subject to an executory interest). In such a case, the estate may end before the life tenant dies if the limiting condition occurs. In this conveyance, A has a life estate subject to a limitation that the premises be used as an animal shelter. Because of the use of durational language ("until"), it can be termed a life estate determinable. Thus, if the premises cease to be used as an animal shelter during A's lifetime, the estate will automatically terminate and revert to O, but A's estate also will terminate at his death, even if the premises are being used as an animal shelter at that time. In a conveyance from O "to A and her heirs," A does not have a fee simple determinable. Rather, A has a fee simple absolute. A fee simple absolute is an estate that has an infinite duration. A fee simple absolute also may be created by a conveyance simply from O "to A."

In general, a life tenant commits permissive waste when he: ASubstantially changes the use of the property in a way that increases its value BFails to make required repairs to the land or pay required carrying charges CConsumes or exploits natural resources on the property

B In general, a life tenant commits permissive waste when he fails to make required repairs to the land or pay required carrying charges (e.g., interest on encumbrances, ordinary taxes). When a life tenant consumes or exploits natural resources on the property (e.g., timber, minerals, oil), he commits voluntary waste. However, exceptions to this rule allow the life tenant to consume or exploit such resources:(i) in reasonable amounts when necessary for repair and maintenance of the land;(ii) when the life tenant is expressly given the right to exploit such resources in the grant (e.g., a mineral lease);(iii) when prior to the grant, the land was used in exploitation of such natural resources, so that the grantor most likely intended the life tenant to have the right to exploit; and(iv) in many states, when the land is suitable only for such exploitation (e.g., a mine). When a life tenant substantially changes the use of the property in a way that increases its value (e.g., he demolishes a run-down building), he commits ameliorative waste.

In most states, when tenants by the entirety divorce, they hold the property as __________. ATenants at will BTenants in common CJoint tenants

B In most states, when tenants by the entirety divorce, they hold the property as tenants in common. A tenancy by the entirety is a marital estate akin to a joint tenancy between husband and wife. Divorce severs a tenancy by the entirety, leaving the parties as tenants in common with no right of survivorship in most states. On the other hand, in a minority of states, when tenants by the entirety divorce, they hold the property as joint tenants with right of survivorship. When tenants by the entirety divorce, they do not hold the property as tenants at will. A tenancy at will is a leasehold estate in land that is terminable at the will of either the landlord or the tenant.

In the absence of a contrary lease provision, which of the following acts would subject a short-term residential tenant to liability for waste? AFailing to repair or replace a worn-out carpet. BRemoving an interior wall to increase the value of the premises. CFailing to replace the roof of the premises. DPumping oil from a well that was active prior to the lease.

B In the absence of a contrary lease provision, removing an interior wall to increase the value of the premises would subject a short-term residential tenant to liability for waste. A tenant is obligated to return the premises in the same condition as at the commencement of the lease term. Thus, a tenant is not permitted to make substantial alterations to the premises even if they increase the value of the property. A tenant who commits waste is liable for the cost of restoration. A modern exception permits changes that increase the value of the premises if they reflect a change in the nature and character of the neighborhood and are performed by a long-term tenant or the holder of a life estate. In that situation, the waste is known as "ameliorative waste." Here, however, the tenant is only a short-term tenant. Pumping oil from a well that was active prior to the lease would not subject a short-term residential tenant to liability for waste. Voluntary waste results when the tenant intentionally or negligently damages the premises. Exploiting minerals on the property also constitutes voluntary waste unless the property was previously so used, as here, or unless the lease provides that the tenant may do so. Failing to replace the roof of the premises would not subject a short-term residential tenant to liability for waste. Permissive waste occurs when the tenant fails to make ordinary repairs to keep the property in the same condition as received. However, unless the lease provides otherwise, the tenant has no duty to make any substantial repairs, such as replacing the roof. Failing to repair or replace a worn-out carpet would not subject a short-term residential tenant to liability for waste. In the absence of a contrary lease provision, a tenant has no duty to repair ordinary wear and tear, such as this.

In the case of a deed, the perpetuities period begins to run on the date the deed is: ARecorded BDelivered with intent to pass title CSigned by the grantor

B In the case of a deed, the perpetuities period begins to run on the date the deed is delivered with intent to pass title. The validity of interests under the Rule Against Perpetuities is determined at the time the interests are created. Because a deed is not effective to transfer an interest in property until it has been delivered with the intent to pass title, the perpetuities period does not begin to run until that time. Thus, the perpetuities period does not begin to run on the date the deed is signed by the grantor or recorded.

May a trespasser remove her annexed chattel from the landowner's property? ANo, unless the chattel increased the value of the land BNo, regardless of whether the chattel was installed in good faith CYes, if the chattel was installed in good faith DYes, regardless of whether the chattel increased the value of the land

B No, a trespasser may not remove her annexed chattel from the landowner's property, regardless of whether the chattel was installed in good faith. A fixture is a chattel that has been so affixed to the realty that it has ceased being personal property and has become part of the realty. A trespasser (e.g., an adverse possessor before the running of the statute of limitations) normally loses her annexations whether or not affixed in good faith and regardless of whether the chattel increased the value of the land. However, if the trespasser acted in good faith and the chattel increased the value of the land, she may recover the value added to the land.

Under the growing trend of modern case law, may a landlord be held liable for defects arising after the tenant takes possession? ANo, because a landlord has no duty to make the premises safe BNo, unless the landlord knew or should have known of them CYes, because a landlord is held to have notice of defects in the premises

B No, under the growing trend of modern case law, a landlord may not be held liable for defects arising after the tenant takes possession unless the landlord knew or should have known of them. At common law, subject to a few exceptions, a landlord had no duty to make the premises safe. However, today, courts are increasingly holding that landlords have a general duty of reasonable care with respect to residential tenants and will be held liable for personal injuries of tenants and their guests resulting from the landlord's ordinary negligence. Even courts that do not go this far still will recognize a landlord's duty of reasonable care in certain limited situations, such as common area defects, latent defects, and defects in premises into which the public is invited. But in all cases, if the defect arises after the tenant takes possession, the landlord is liable only if the landlord knew or had reason to know of the defect. Although a landlord is held to have notice of defects in the premises that existed before the tenant took possession, a landlord may not be liable for defects arising after the tenant takes possession unless there is evidence that the landlord actually knew or should have known of them.

The Rule in Dumpor's Case provides that: AA covenant prohibiting assignment prohibits subleasing, and vice versa BIf a landlord consents to one transfer that would otherwise violate a covenant against assignment or sublease, he waives his right to assert that future transfers breach the lease CA covenant against assignment or sublease is void as an unreasonable restraint on alienation DIf a tenant transfers her interest in violation of a covenant against assignment or sublease, the transfer is void

B The Rule in Dumpor's Case provides that if a landlord consents to one transfer that would otherwise violate a covenant against assignment or sublease, he waives his right to assert that future transfers breach the lease. The landlord must expressly reserve the right to object to future transfers at the time of granting consent to the initial transfer. A covenant prohibiting assignment does NOT prohibit subleasing, and vice versa. Covenants against assignment or sublease are strictly construed against the landlord, and absent an express restriction in the lease, a tenant may freely transfer his leasehold interest, in whole or in part. A complete transfer of the tenant's entire remaining term is an assignment, and a transfer retaining any part thereof is a sublease. A prohibition of either type of transfer does not prohibit the other type. If a tenant transfers her interest in violation of a covenant against assignment or sublease, the transfer is NOT void. However, the landlord usually may terminate the lease under the lease terms or a statute or sue for damages. A covenant against assignment or sublease is NOT void as an unreasonable restraint on alienation. All jurisdictions enforce such covenants.

Which of the following acts will not sever a tenancy by the entirety? ADeath of one spouse BExecution by a creditor of one spouse CDivorce of the spouses DMutual agreement of the spouses

B The execution by a creditor of one spouse will not sever a tenancy by the entirety. A tenancy by the entirety is a marital estate akin to a joint tenancy between husband and wife. Severance of a tenancy by the entirety is limited. Although a joint creditor of both spouses can execute a judgment against the property, terminating the tenancy by the entirety, a creditor of only one spouse cannot. The death of one spouse will sever a tenancy by the entirety, leaving the surviving spouse as sole owner of the property. The divorce of the spouses will sever a tenancy by the entirety, leaving the parties as tenants in common with no right of survivorship in most states. The mutual agreement of the spouses (e.g., deed executed by both spouses) will sever a tenancy by the entirety. A deed or mortgage executed by only one spouse, however, is ineffective to cause a severance

The three major types of leasehold estates in land are: ATenancies at sufferance, hold-over tenancies, and implied tenancies BTenancies for years, periodic tenancies, and tenancies at will CJoint tenancies, tenancies in common, and tenancies by the entirety

B The three major types of leasehold estates in land are tenancies for years, periodic tenancies, and tenancies at will. A tenancy for years is a leasehold estate that continues for a fixed period of time and then automatically expires. A periodic tenancy is a leasehold estate that continues from period to period until terminated by either the landlord or the tenant giving proper notice. A tenancy at will is a leasehold estate that continues only until the landlord or the tenant gives notice and time to quit. Tenancies at sufferance, hold-over tenancies, and implied tenancies are not the three major types of leasehold estates in land. A tenancy at sufferance arises when a tenant wrongfully remains in possession (i.e., "holds over") after the expiration of a lawful tenancy. Tenancies at sufferance are considered by some authorities not to be estates in land, as the tenant remains in possession only until the landlord takes steps to evict him. Alternatively, the landlord may choose to accept rent and bind the hold-over tenant to a new periodic tenancy. While this creates a periodic tenancy by operation of law, periodic tenancies also can be implied from a lease that states no definite time period but provides for periodic rent payments. However, the type of leasehold estate created is a periodic tenancy. Joint tenancies, tenancies in common, and tenancies by the entirety are not leasehold estates in land. They are forms of concurrent ownership in land, wherein multiple persons have the right to the enjoyment and possession of the land at the same time.

Under the growing trend of modern case law, a landlord will be held liable for personal injuries of residential tenants and their guests resulting from the landlord's __________ negligence. Acomparative Bordinary Cslight Dcontributory

B Under the growing trend of modern case law, a landlord will be held liable for personal injuries of residential tenants and their guests resulting from the landlord's ordinary negligence. This duty of reasonable care is ordinarily not imposed until the landlord has notice of a particular defect and a reasonable opportunity to repair it. Slight negligence involves the failure to exercise great care. A landlord generally will not be held to this high standard of care. Contributory and comparative negligence are defenses to negligence. At common law, a plaintiff's contributory negligence completely barred his right to recover. Under a comparative negligence system, a contributorily negligent plaintiff may recover a percentage of his damages.

What is a landlord's duty with respect to common areas that remain under his control? AThe landlord must exercise extraordinary care BThe landlord must exercise reasonable care CThe landlord must exercise slight care DThe landlord has no duty to make the premises safe

B With respect to common areas that remain under his control (e.g., hallways, elevators, etc.), a landlord generally has a duty to exercise reasonable care. The landlord is liable for any injury resulting from a dangerous condition that reasonably could have been discovered and made safe. This is the same duty an owner-occupier owes his guests under tort law, not extraordinary care or slight care. At common law, subject to a few exceptions, a landlord had no duty to make the premises safe. However, several exceptions exist today, including the rule for common areas, discussed above.

May a tenant remove a chattel that the tenant affixed to the leased premises? ANo, because chattels affixed to the leased premises become the property of the landlord BYes, if removal occurs before termination of the lease and leaves no damage to the premises CYes, because chattels affixed to the leased premises remain the property of the tenant DNo, unless the landlord and tenant expressly agreed that the chattel would be considered a fixture

B Yes, absent an agreement to the contrary, a tenant may remove a chattel that the tenant affixed to the leased premises if removal occurs before termination of the lease and leaves no damage to the premises. A fixture is a chattel that has been so affixed to the realty that it has ceased being personal property and has become part of the realty. At early common law, chattels affixed to the leased premises became the property of the landlord and thus could not be removed from the premises by the tenant. Today, a tenant may remove annexed chattels before the termination of the tenancy if doing so causes no damage to the premises (or any damage done by removing them is repaired by the tenant). Chattels affixed to the leased premises do NOT always remain the property of the tenant. Chattels the tenant affixes to leased premises may become the landlord's property (i.e., fixtures) under certain circumstances, such as where the tenant fails to remove the chattels before the end of the lease term, removal of the chattels will substantially damage the premises, or the landlord and tenant agreed that the chattels were intended to become fixtures. A tenant may NOT remove an affixed chattel if the landlord and tenant expressly agreed that the chattel would be considered a fixture. An agreement between the landlord and tenant is controlling as to whether the chattel annexed to the premises was intended to become a fixture. Thus, if the landlord and tenant agree that an annexation is not a fixture, the tenant will be permitted to remove it before the end of the lease term. However, if the landlord and tenant agree that an annexation is a fixture, it becomes the landlord's property.

A leasehold is: ANot an estate in land BA freehold estate in land CA nonfreehold estate in land

C A leasehold is a nonfreehold estate in land. Nonfreehold estates in land (leaseholds) give possession of land for a limited time. The tenant has a present possessory interest in the leased premises, and the landlord has a future interest (reversion). A leasehold is not a freehold estate in land. Freehold estates in land (e.g., fee estates and life estates), like leaseholds, give possession, but their duration is regarded as longer than that of the nonfreehold (or leasehold) estates. A leasehold IS an estate in land. Possessory interests in land are estates in land, and as explained above, both the tenant and the landlord have possessory interests in leased premises. In contrast, nonpossessory interests in land (e.g., easements, profits, and covenants) are not estates in land

Which of the following is a life tenant obligated to perform? AObtain insurance on the land. BMake permanent improvements to the land. CPay ordinary taxes on the land. DPay the principal of a mortgage on the land.

C A life tenant is obligated to pay ordinary taxes on the land, provided that they do not exceed the rent or fair rental value of the land. Failure to pay the taxes constitutes permissive waste and subjects the life tenant to liability for damages. Although a life tenant is obligated to make reasonable repairs to the land and structures, he is not obligated to make permanent improvements to the land. Although a life tenant is obligated to pay the interest on a mortgage on the land, he is not obligated to pay the principal of a mortgage on the land. However, a life tenant who places a mortgage solely on his life estate is liable for both principal and interest payments. A life tenant is not obligated to obtain insurance on the land, but he may do so.

A tenancy by the entirety can be terminated by: AExecution by a creditor of one spouse BSeparation of the spouses CMutual agreement of the spouses

C A tenancy by the entirety can be terminated by mutual agreement of the spouses (e.g., a deed executed by both spouses). A tenancy by the entirety is a marital estate akin to a joint tenancy between husband and wife. It can be terminated only by:(i) the death of either spouse (leaving the surviving spouse as sole owner of the property);(ii) divorce (leaving the parties as tenants in common with no right of survivorship in most states);(iii) mutual agreement; or(iv) execution by a joint creditor of both spouses. Mere separation of the spouses (as opposed to a divorce) or execution by a creditor of one spouse does not terminate a tenancy by the entirety.

Which of the following acts will sever a joint tenancy? AA testamentary disposition by one joint tenant BIn a lien theory state, the execution of a mortgage by one joint tenant CAn inter vivos conveyance by one joint tenant

C An inter vivos conveyance by one joint tenant will sever a joint tenancy. The grantee then becomes a tenant in common with the original joint tenant or tenants. This applies to both voluntary and involuntary conveyances; the typical example is a deed. In a lien theory state, the execution of a mortgage by one joint tenant will not sever a joint tenancy. In states following the lien theory, a mortgage is regarded as a lien on title, and one joint tenant's execution of a mortgage on her interest does not by itself cause a severance. But in states following the title theory, a mortgage is regarded as a transfer of title, and the transfer destroys the unity of title and severs the joint tenancy. A testamentary disposition by one joint tenant will not sever a joint tenancy. A will devising a joint tenant's interest to another is inoperative as to joint tenancy property because when the co-tenant who is the testator dies (which is when the will becomes effective), his rights in the joint tenancy property are extinguished, and the will has no effect on them.

If A, B, and C own property as joint tenants, and P obtains a money judgment against A, what interests do the parties own on B's death? AC owns a 2/3 interest, and P owns a 1/3 interest BB's estate, C, and P each own a 1/3 interest CA owns a 1/2 interest subject to P's lien, and C owns a 1/2 interest

C If A, B, and C own property as joint tenants, and P obtains a money judgment against A, on B's death A owns a 1/2 interest subject to P's lien, and C owns a 1/2 interest. In most states, a judgment lien obtained against one of several joint tenants does not sever the joint tenancy because it is not considered a sufficiently substantial "conveyance" to destroy the unities of time and title. However, if the plaintiff who obtained the judgment proceeds to enforce it by foreclosure, the sheriff's deed issued to the buyer at that sale will sever the joint tenancy. Because P has not enforced her lien against A by foreclosure, A retains his interest as a joint tenant, but his interest is subject to P's lien. When one joint tenant dies, the property is freed from her interest, and the survivors retain an undivided right in the property. Since B's interest was extinguished on her death, B's estate does not take B's interest; the surviving joint tenants (A and C) hold free of it. C does NOT own a 2/3 interest, and P does NOT own a 1/3 interest. This would be the case if P's judgment lien severed the joint tenancy. When property is held by three or more joint tenants, one joint tenant's conveyance destroys the joint tenancy only as to that interest. The remaining joint tenants continue to hold in joint tenancy as between themselves, and the grantee holds her interest as a tenant in common with them. If P's judgment lien severed A's 1/3 interest, that interest would have been converted into a tenancy in common, which P would continue to hold. Thus, C would own a 2/3 interest in the property on B's death. B's estate, C, and P do NOT each own a 1/3 interest. As is explained above, B's estate does not take B's interest on B's death, and P's judgment lien did not sever the joint tenancy.

If O conveys property "to A for her support until she remarries," what interest does A have? AFee simple absolute BFee simple subject to a condition subsequent CFee simple determinable DFee simple subject to an executory interest

C If O conveys property "to A for her support until she remarries," A has a fee simple determinable. A fee simple determinable is an estate that automatically terminates on the happening of a stated event and reverts to the grantor. It is created by the use of durational, adverbial language, such as "for so long as," "while," "during," or "until." A has a fee simple because the estate may last forever if she never remarries. If, however, A remarries, the estate will automatically terminate and possession of the land will revert to O. A does not have a fee simple absolute. A fee simple absolute is the largest real property estate permitted by law. It has an infinite duration and is not susceptible to being terminated early by any event. A's estate will terminate upon the happening of a stated event (A's remarriage). Thus, A has a defeasible fee: a fee simple determinable, as explained above. Although restraints intended to penalize marriage or encourage divorce (e.g., "to A if she divorces B") may be struck down as violations of public policy, restraints intended to provide support until marriage generally are upheld. Because O's purpose is to provide support to A until she remarries, the restraint is valid. A does not have a fee simple subject to a condition subsequent. A fee simple subject to a condition subsequent is an estate that terminates when the grantor exercises her power of termination (right of entry) after the happening of a stated event. Words such as "upon condition that," "provided that," "but if," and "if it happens that" are usually held to create conditions subsequent, and under most decisions the grantor must expressly reserve the right of entry. Here, O will immediately and automatically become the owner of the fee simple upon A's remarriage, without taking any steps to terminate A's interest. A does not have a fee simple subject to an executory interest. A fee simple subject to an executory interest is an estate that, on the happening of a stated event, automatically divests in favor of a third person rather than the grantor. Upon A's remarriage, O, the grantor, rather than a third person, will automatically become the owner of the fee simple.

If O conveys property "to charity A, but if the premises are no longer used as a museum, to charity B," charity A has __________ and charity B has __________. Aa fee simple determinable; a possibility of reverter Ba fee simple on condition subsequent; a right of entry Ca fee simple subject to an executory interest; an executory interest Da fee simple absolute; no interest

C If O conveys property "to charity A, but if the premises are no longer used as a museum, to charity B," charity A has a fee simple subject to an executory interest and charity B has an executory interest. A fee simple subject to an executory interest is an estate that, on the happening of a stated event, is automatically divested in favor of a third person rather than the grantor. The words "but if the premises are no longer used as a museum" create a condition on which charity A's interest will be terminated and a third party, charity B, will get possession of the land; hence charity A's estate is subject to an executory interest in favor of charity B, which would be referred to simply as a fee simple subject to an executory interest. Charity B is not the grantor, and thus its interest is not a right of entry (the correlative future interest in a grantor who conveys a fee simple subject to a condition subsequent). Its interest is an executory interest because it divests a fee simple. Although executory interests are subject to the Rule Against Perpetuities, charity B's interest does not violate the Rule because the charity-to-charity exception provides that a gift over from one charity to another charity on a remote condition is valid. Charity A does not have a fee simple determinable and charity B does not have a possibility of reverter. A fee simple determinable is an estate that automatically terminates on the happening of a stated event and reverts to the grantor. It is created by the use of durational, adverbial language, such as "for so long as," "while," "during," or "until." A possibility of reverter arises only in the grantor. If a comparable interest is created in a third party, it is an executory interest. Charity A's interest is not properly considered a fee simple on condition subsequent; if it were, then the grantor would have a right of entry and could take back the land upon the occurrence of the condition. When a third party, such as charity B, gets possession upon the happening of the condition, the initial grantee's estate is called a fee simple subject to an executory interest. Moreover, charity B does not have a right of entry because it is not the grantor; a similar interest created in favor of a third party is called an executory interest. Charity A does not have a fee simple absolute and charity B does not have no interest. The Rule Against Perpetuities provides that certain interests in property are void if there is any possibility, however remote, that they may vest more than 21 years after some life in being at the creation of the interest. If charity B's interest had violated the Rule, its interest would have been stricken, leaving charity A with a fee simple absolute ("to charity A"). However, as explained above, charity B's interest is valid under the charity-to-charity exception to the Rule.

Which statement is true regarding tenancies at will? ABoth the landlord's and the tenant's rights to terminate must be express and will not be implied BIf a lease gives only the tenant the right to terminate at will, a similar right generally will be implied in favor of the landlord CIf a lease gives only the landlord the right to terminate at will, a similar right generally will be implied in favor of the tenant DIf a lease gives only the landlord the right to terminate at will, a similar right generally will not be implied in favor of the tenant

C If a lease gives only the landlord the right to terminate at will, a similar right generally will be implied in favor of the tenant. A tenancy at will is a tenancy that continues only until the landlord or the tenant gives notice and time to quit. Because in a tenancy at will both the landlord and the tenant must have the right to terminate the lease at will, courts usually imply a right to terminate in favor of the tenant if the lease gives only the landlord the right to terminate at will. On the other hand, if a lease gives only the tenant the right to terminate at will, a similar right generally will NOT be implied in favor of the landlord. Rather, most courts would interpret the conveyance as creating a life estate or fee simple terminable by the tenant (i.e., determinable). However, it is not true that both the landlord's and the tenant's rights to terminate must be express and will not be implied. As explained above, if the lease gives the landlord such a right, generally a court will imply the same for the tenant.

Which of the following is not a modern remedy available to most landlords when a tenant defaults on her rent obligation? ATerminate the lease and evict the tenant under the jurisdiction's unlawful detainer statute. BDeduct the unpaid rent from the tenant's security deposit. CEnter the leased premises without notice and seize the tenant's chattels as security for the unpaid rent. DBring a cause of action against the tenant for damages.

C If a tenant defaults on her rent obligation, under modern law in most states, a landlord may not enter the leased premises without notice and seize the tenant's chattels as security for the unpaid rent. At common law, this was known as the right of distress. Some states have abolished this common law right, while others have enacted distress statutes that eliminate the self-help feature (seizure by the landlord) or require prior notice and a hearing before property is taken. The common law landlord right of self-help distress is now a minority rule. On the other hand, if a tenant defaults on her rent obligation, a landlord may: Deduct the unpaid rent from the tenant's security deposit;Bring a cause of action against the tenant for damages; or Terminate the lease and evict the tenant under the jurisdiction's unlawful detainer statute

If the government condemns __________ of the leased land, the tenant's liability for rent __________, and the tenant __________ entitled to compensation for the taking. Apart; is apportioned; is not Ball; continues; may be Call; is extinguished; may be Dpart; is extinguished; is

C If the government condemns all of the leased land, the tenant's liability for rent is extinguished, and the tenant may be entitled to compensation for the taking. Because both the leasehold and the reversion merge in the condemnor, the lease is terminated. Thus, the tenant's rent liability does NOT continue. If the government condemns part of the leased land, the tenant's liability for rent is NOT extinguished. The tenant may be entitled to have rent apportioned and may be entitled to compensation for the taking.

Which of the following transfers creates a sublease from T to T2? ASix months into a seven-month tenancy for years, T transfers his interest "to A for the balance of the leasehold term" BOne year into a five-year tenancy for years, T transfers his interest "to T2 for four years; however, if T2 breaches the original lease terms, T may reenter and retake the premises" CTwo years into a four-year tenancy for years, T "assigns my entire interest to T2 for one year"

C If two years into a four-year tenancy for years, T "assigns my entire interest to T2 for one year," the effect of the transfer is to create a sublease between T and T2. The label given by the parties does not determine whether a transfer is an assignment or a sublease. Rather, a complete transfer of a tenant's entire remaining lease term is an assignment, and a transfer retaining any part thereof is a sublease. Here, although T "assigned" his interest to T2, he transferred only one of the remaining two years of the lease. Thus, the transfer is a sublease rather than an assignment. If six months into a seven-month tenancy for years, T transfers his interest "to A for the balance of the leasehold term," the effect of the transfer is an assignment of the lease from T to T2 because it includes T's entire remaining lease term. If one year into a five-year tenancy for years, T transfers his interest "to T2 for four years; however, if T2 breaches the original lease terms, T may reenter and retake the premises," the effect of the transfer is an assignment of the lease from T to T2. T transferred the remaining four years of the lease to T2, and by the slight majority view, T's reservation of a right of reentry does not result in a sublease, but rather is still an assignment.

In residential leases, a tenant who holds over after termination of a year-to-year periodic tenancy may be held to a new periodic tenancy from __________. AWeek to week BYear to year, with rent payable annually CMonth to month DYear to year, with rent payable monthly

C In residential leases, a tenant who holds over after termination of a year-to-year periodic tenancy may be held to a new periodic tenancy from month to month. If the tenant continues in possession after the termination of a lawful tenancy, the landlord may evict the tenant or bind the tenant to a new periodic tenancy. The terms and conditions of the expired tenancy (e.g., rent, covenants, etc.) apply to the new tenancy. Most courts would rule that a residential hold-over tenant becomes a month-to-month periodic tenant, regardless of the term of the original lease. Thus, even though the lease was for one year or more, the tenant may not be held to a new periodic tenancy from year to year, whether rent were payable monthly or payable annually. In contrast, a hold-over tenant under a commercial lease for one year or more may be held to a new year-to-year periodic tenancy. In residential leases, a tenant who holds over under a year-to-year periodic tenancy probably will not be held to a new periodic tenancy from week to week. While this might occur if the residential tenant were a roomer paying weekly rent, facts to that effect are not present here.

L leases property to T, and T covenants to pay rent. T later transfers the balance of the leasehold term to T2. Whom may L hold liable for the subsequent failure to pay rent? AOnly T BOnly T2 CEither T or T2

C L may hold either T or T2 liable for the subsequent failure to pay rent. A complete transfer of a tenant's entire remaining lease term is an assignment of the lease. The assignee and the landlord are in privity of estate, with each liable to the other on all lease covenants that run with the land (e.g., the covenant to pay rent). Thus, an assignee owes rent directly to the landlord. After assignment, the original tenant is no longer in privity of estate with the landlord, but remains in privity of contract. Because T promised to pay rent in his lease with L, he can still be held liable on his original contractual obligation to pay. Thus, L may hold liable the original tenant (T) or the assignee (T2) for failure to pay rent. If, by contrast, the transfer had been a sublease rather than an assignment, L could have held only T liable for failure to pay rent. A tenant's transfer of less than the entire remaining lease term is a sublease. The sublessee is liable to the original tenant for rent as agreed to in the sublease, but is not personally liable to the landlord for rent or any covenants made by the original tenant in the original lease. This is because the sublessee neither has a contractual relationship with the landlord (i.e., no privity of contract) nor holds the tenant's full estate in the land (i.e., no privity of estate). Thus, the covenants in the original lease do not run with the land to bind the sublessee. Note that while, in a sublease, T2 is not personally liable to L for the rent, T2 still has a strong interest in ensuring that the rent is paid, because if it is not, L has the right to terminate the lease, thus eliminating T2's right to possess the premises.

Must a co-tenant in possession share profits gained by her personal use of the property with co-tenants out of possession? AYes, because such use constitutes an ouster BNo, if the use is to rent out the property to a third party CNo, unless there has been an ouster or agreement to the contrary

C No, a co-tenant in possession need not share profits gained by her personal use of the property with co-tenants out of possession unless there has been an ouster or agreement to the contrary. An ouster occurs when one co-tenant wrongfully excludes another co-tenant from possession of the whole or any part of the whole of the property. A claim of right to exclusive possession can constitute an ouster, but mere personal use of the property in a manner that benefits or generates profits for the co-tenant in possession does NOT constitute an ouster. However, if the co-tenant in possession rents out the property to a third party, she must account to the co-tenant out of possession for his share of the net rents (after deducting operating expenses for the rental property).

May a tenant waive the implied warranty of habitability? AYes, if the tenant accepts the premises "as is" BYes, if the tenant covenants to repair CNo, because such a waiver is against public policy

C No, a tenant may not waive the implied warranty of habitability, because such a waiver is against public policy. Under the implied warranty of habitability, a landlord assures that the premises are suitable for human residence. The standard usually applied is the local housing code. One of the reasons for implying a covenant of habitability is to encourage enforcement of the housing code by tenants. Thus, even if the tenant accepts the premises "as is" or covenants to repair, the landlord's obligations under the implied warranty of habitability are usually held to be nonwaivable.

Which of the following interests in property are not subject to the Rule Against Perpetuities? ARights of first refusal BContingent remainders CReversions DExecutory interests

C Reversions are not subject to the Rule Against Perpetuities. The Rule Against Perpetuities provides that certain interests in property are void if there is any possibility, however remote, that they may vest more than 21 years after some life in being at the creation of the interest. The Rule applies to the following interests in property: (i) contingent remainders, (ii) executory interests, (iii) class gifts (even if vested remainders), (iv) options and rights of first refusal, and (v) powers of appointment. Future interests in the grantor (i.e., reversions, possibilities of reverter, and rights of entry) are not subject to the Rule.

A tenant leases premises that are destroyed without the fault of either the landlord or the tenant. Under the majority view, which is true in the absence of a contrary lease provision? AThe lease remains effective, and the tenant may cease paying rent BThe lease remains effective, and the tenant must continue paying rent CThe lease may be terminated at the tenant's option, and the tenant may cease paying rent

C Under the majority view, in the absence of a contrary lease provision, if the leased premises are destroyed without the fault of either the landlord or the tenant, the lease may be terminated at the tenant's option, and the tenant may cease paying rent. The common law rule differs from the majority view in that if the leased premises are destroyed without the fault of either the landlord or the tenant, the lease remains effective and the tenant must continue paying rent. Note that the harsh common law rule has been modified by statute in most states. Absent a contrary lease provision, if under applicable law the lease remains effective after destruction of the leased premises without the fault of either the landlord or the tenant, the tenant may NOT cease paying rent.

Which of the following is not a type of leasehold estate? APeriodic tenancy BTenancy for years CTenancy at will DTenancy in common

D A tenancy in common is not a type of leasehold estate. A leasehold estate is a property relationship between landlord and tenant in which the tenant has a present possessory interest in the leased premises, and the landlord has a future interest (reversion). A tenancy in common, on the other hand, is a concurrent estate in land in which each owner has a fractional or percentage undivided interest in the property. It provides no right of survivorship. A tenancy for years is a leasehold estate that continues for a fixed period of time and automatically expires without either the landlord or the tenant giving notice to the other. A periodic tenancy is a leasehold estate that continues from period to period until terminated by proper notice by either the landlord or the tenant. A tenancy at will is a leasehold estate that is terminable at the will of either the landlord or the tenant.

When property is held in joint tenancy or tenancy in common, which of the following is not a co-tenant's right? APossess the entire estate BMortgage her interest CShare in rents paid by third parties DCompel contribution for the cost of improvements

D Although a joint tenant or tenant in common may have a right to compel contribution from other co-tenants for the cost of necessary repairs, taxes, and payments due on mortgages, she does not have a right to compel contribution for the cost of improvements. Under the unity of possession, each co-tenant has a right to possess the entire estate subject to the equal right of her co-tenant. A co-tenant out of possession cannot bring a possessory action unless there has been an "ouster" (i.e., wrongful exclusion) by the co-tenant in possession. Although a co-tenant generally is not entitled to share in the rental value of the land, she does have a right to share in rents paid by third parties. A joint tenant or tenant in common may mortgage her interest. However, she may not encumber another co-tenant's interest. Note that an individual spouse may not mortgage her interest in tenancy by the entirety property.

Which of the following interests in property are subject to the Rule Against Perpetuities? AContingent remainders, possibilities of reverter, and rights of first refusal BExecutory interests, rights of entry, and powers of appointment CReversions, options, and class gifts DContingent remainders, executory interests, and vested remainders subject to open

D Contingent remainders, executory interests, and vested remainders subject to open are subject to the Rule Against Perpetuities. The Rule Against Perpetuities provides that certain interests in property are void if there is any possibility, however remote, that they may vest more than 21 years after some life in being at the creation of the interest. The Rule applies to the following interests in property:(i) contingent remainders;(ii) executory interests;(iii) class gifts (even if vested remainders);(iv) options and rights of first refusal; and(v) powers of appointment. Future interests in the grantor (i.e., reversions, possibilities of reverter, and rights of entry) are not subject to the Rule Against Perpetuities. Thus, contingent remainder, possibilities of reverter, and rights of first refusal is incorrect because it includes possibilities of reverter. Executory interests, rights of entry, and powers of appointment is incorrect because it includes rights of entry. Reversions, options, and class gifts is incorrect because it includes reversions.

If A and B own property as joint tenants, and B dies leaving a will devising her interest in the property to C, who owns the property? AC only BA and C, as joint tenants CA and C, as tenants in common DA only

D If A and B own property as joint tenants, and B dies leaving a will devising her interest in the property to C, A only owns the property. A testamentary disposition by one joint tenant will not sever a joint tenancy. A will devising a joint tenant's interest to another is inoperative as to joint tenancy property because when the co-tenant who is the testator dies (which is when the will becomes effective), her rights in the joint tenancy property are extinguished, and the will has no effect on them. Thus, upon B's death the property is freed from her concurrent interest, leaving A the sole owner and C with no interest in the property. On the other hand, certain acts by one joint tenant will sever a joint tenancy (e.g., suit for partition, inter vivos conveyance by one joint tenant, execution of a mortgage by one joint tenant in a title theory state). Then, the transferee takes the interest as a tenant in common and not as a joint tenant. Thus, if B had successfully conveyed her interest to C by deed, A and C would own the property as tenants in common but not as joint tenants. Alternatively, if A and B had owned the property as tenants in common, B's will would have effectively conveyed her interest to C, so that A and C would own the property as tenants in common. Furthermore, C only would be incorrect in any event because B can convey no greater interest than the one-half interest she owns.

If L leases a residence to T for five years, ending on December 31, at $400 per month, and T remains in possession of the premises on January 1 of the sixth year, L may hold T to a __________. Ayear-to-year tenancy and may now demand $500 per month Byear-to-year tenancy at $400 per month Cmonth-to-month tenancy and may now demand $500 per month Dmonth-to-month tenancy at $400 per month

D If L leases a residence to T for five years, ending on December 31, at $400 per month, and T remains in possession of the premises on January 1 of the sixth year, L may hold T to a month-to-month tenancy at $400 per month. If a tenant continues in possession after the termination of a lawful tenancy, the landlord may evict the tenant or bind the tenant to a new periodic tenancy. The terms and conditions of the expired tenancy (e.g., rent, covenants, etc.) apply to the new tenancy. Although a commercial tenant holding over after termination of a lease for one year or more generally may be held to a year-to-year tenancy, most courts would rule a residential hold-over tenant a month-to-month tenant, irrespective of the term of the original lease. If the landlord notifies the tenant before termination of the tenancy that occupancy after termination will be at an increased rent, the tenant will be held to the new rent if he does not surrender. Here, T's five-year tenancy for years automatically expired on December 31. Thus, on January 1 he was a hold-over tenant and L could bind him to a new periodic tenancy. As a residential tenant, the period would be month to month rather than year to year. Because there is no evidence that L informed T of the rent increase prior to December 31, T will be liable only for $400 per month. L may not hold T to a month-to-month tenancy and now demand $500 per month because L did not notify T of the rent increase prior to the expiration of his five-year tenancy. L may not hold T to a year-to-year tenancy at $400 per month because T is a residential rather than a commercial tenant. L may not hold T to a year-to-year tenancy and now demand $500 per month because T is a residential rather than a commercial tenant, and because L did not notify T of the rent increase prior to the expiration of his five-year tenancy.

If T devises property "to A for life, then to B's children," and B has one child (W) on the date the will is executed, two children (W and X) at T's death, three children (W, X, and Y) at A's death, and four children (W, X, Y, and Z) at B's death, who will share in the class gift? AW, X, Y, and Z BW only CW and X only DW, X, and Y only

D If T devises property "to A for life, then to B's children," and B has one child (W) on the date the will is executed, two children (W and X) at T's death, three children (W, X, and Y) at A's death, and four children (W, X, Y, and Z) at B's death, W, X, and Y only will share in the class gift. Under the rule of convenience, a class closes when some member of the class can call for a distribution of her share of the gift. When possession and enjoyment of a gift are postponed, as where the gift follows a life estate, the class remains open until the time fixed for distribution (e.g., death of the life tenant). At T's death, the will became effective and A's life estate began. At that time, W and X had a vested remainder subject to open because their interest was certain to take on the termination of A's life estate but also could be partially divested by B's unborn children. At A's death, W, X, and then-born Y were entitled to distribution of the property, so the class closed at that time. Z is excluded by the rule of convenience. W only is incorrect because the class did not close on the date the will was executed. Because a will is not effective until the testator dies, the beneficiaries have only an expectancy and not an actual interest in the property until that time. W and X only is incorrect because the class did not close at T's death. When a will makes an outright gift of a present interest to a class, if any class members are alive at the testator's death, the class closes on the date of the testator's death. If the devise had been "to B's children" without A's preceding life estate, the class would have closed at T's death and only W and X would have been entitled to share in the class gift. Because the class gift was a future interest to take possession upon A's death, however, Y also is entitled to a share. W, X, Y, and Z is incorrect because the class closed at A's death rather than B's. Under the rule of convenience, distributing one-third shares to W, X, and Y at A's death, but later requiring them to make rebates as B had more children, would be inconvenient. Thus, Z is excluded from sharing in the gift.

If a landlord's breach of duty renders the premises unsuitable for occupancy, the tenant may: ARemain in possession of the premises, continue to pay rent, and sue for damages BRemain in possession of the premises and refuse to pay rent until the interference ceases CSue for breach only if the lease contained an express covenant for quiet enjoyment DVacate the premises, terminate the lease, and sue for damages

D If a landlord's breach of duty renders the premises unsuitable for occupancy, the tenant may vacate the premises, terminate the lease, and sue for damages. Under the doctrine of constructive eviction, if the landlord's breach (i.e., doing an act or failing to provide some service that he has a legal duty to provide) makes the premises untenantable, the tenant may terminate the lease and also may seek damages if the following conditions are met:1. The breach must be by the landlord or by persons acting for him.2. The breach must substantially and materially deprive the tenant of her use and enjoyment of the premises (e.g., flooding, absence of heat in winter).3. The tenant must give the landlord notice and a reasonable time to repair.4. The tenant must vacate the premises within a reasonable time. Because a tenant cannot claim a constructive eviction unless and until she vacates the premises, her remedies do not include remaining in possession of the premises and refusing to pay rent until the interference ceases or continuing to pay rent and suing for damages. The tenant is not limited to suing for breach only if the lease contained an express covenant for quiet enjoyment. Every lease contains an implied covenant that neither the landlord nor someone with paramount title will interfere with the tenant's quiet enjoyment and possession of the premises. If a landlord does so, the tenant has the remedies discussed above.

In the case of a deed, the perpetuities period begins to run on the date: AThe deed is acknowledged before a notary BThe deed is signed by the grantor CThe deed is signed by the grantee DThe deed is delivered with the intent to pass title

D In the case of a deed, the perpetuities period begins to run on the date the deed is delivered with the intent to pass title. The validity of interests under the Rule Against Perpetuities is determined at the time the interests are created. For a deed, this is the date of delivery, not the date the deed is signed by the grantor or the grantee. In the case of a deed, the perpetuities period does not necessarily begin to run on the date the deed is acknowledged before a notary. Delivery is presumed if a deed is both acknowledged by the grantor and recorded, but even so, this presumption is rebuttable

Life tenants are obligated to make which payments on a mortgage on the land? AOnly the principal BNeither the principal nor the interest CThe principal and the interest DOnly the interest

D Life tenants are obligated to make only the interest payments on a mortgage on the land. Failure to pay interest on any encumbrances on the land constitutes permissive waste. Principal payments, if any, are the responsibility of the holder of the future interest (typically a remainder or reversion) that follows the life estate. Although life tenants are obligated to pay the interest on a mortgage on the land, they are not obligated to pay the principal of a mortgage on the land. Thus, the answer choices requiring the life tenants to pay only the principal and both the principal and the interest are incorrect. But note: A life tenant who places a mortgage solely on his life estate is liable for both the principal and the interest payments.

One week after a month-to-month tenant reports her landlord for housing code violations, the landlord gives the tenant 30 days' notice that he is terminating her lease. May the landlord evict the tenant after 30 days? AYes, because the tenant may seek damages under the implied warranty of habitability BYes, because one month's notice is required to terminate a month-to-month tenancy CYes, unless the tenant can prove the landlord had a retaliatory motive DNo, unless the landlord can show a valid, nonretaliatory motive

D No, the landlord may not evict the tenant after 30 days unless the landlord can show a valid, nonretaliatory motive. A landlord may not penalize a tenant for exercising her legal right to report housing or building code violations. Retaliatory acts include raising the rent; reducing tenant services; and terminating the lease, even as to periodic tenants to whom the landlord gives notice. Generally, retaliatory eviction statutes presume retaliation if the landlord acts shortly after the tenant exercises her rights (within 90 to 180 days in many states). Because retaliation is presumed, the tenant would be protected even if the tenant could not prove the landlord had a retaliatory motive. However, the landlord may overcome the presumption as stated above. Although one month's notice is required to terminate a month-to-month tenancy, the tenant could claim retaliatory eviction because her lease was terminated shortly after she reported the housing code violations, as explained above. Under the implied warranty of habitability, the landlord covenants that the premises are suitable for human residence (e.g., up to the standards of the local housing code). Although the tenant may seek damages against a landlord who violates this warranty, a tenant who reports housing code violations may not be evicted in retaliation.


संबंधित स्टडी सेट्स

English Civil War and American Revolution

View Set